Решить задача по физике: Решение задач по 📝 физике быстро и качественно без посредников

Содержание

Как решать задачи по физике легко

Блог

Главная

Блог

Как решать задачи по физике

23.09.2019

15:35

Предмет «Физика» дается с легкостью не каждому ученику. Некоторые задачи вызывают изумления и трудности даже у преподавателей. Существует несложная система, придерживаясь которой решить любую задачу будет реально, сомневаться в ответе не придется, а у педагога отпадут поводы «придраться». И подготовка к ЕГЭ по физике будет даваться легче.

Решение задач по физике без затруднений

Перед тем как приступить к мозговому штурму, стоит успокоиться и вдумчиво прочитать задачу полностью. Иногда она может быть простой, однако непонятные слова могут создать впечатление безвыходности. Ознакомившись, стоит перечитать еще раз. Вникли? Записываем условия.

Записываем условия

  1. Пишем «дано». С этого шага начинается решение любой задачи. В этом блоке записываем все известные условия, чтобы с легкостью можно было к ним обратиться.
  2. Таблица «СИ». Сведения вписываются справа от «Дано», требуются, когда нужно перевести какое-то значение (например, сантиметры в метры).

Рисуем схему

Большинство задач подразумевает наличие схемы, даже если этот шаг необязателен, составление схемы облегчит их понимание. Рисунок должен содержать все известные величины, требующиеся для решения. Иногда за добровольное включение схемы начисляются дополнительные баллы к решению задачи.

Определяем неизвестные величины

  • Вопрос задачи. Выписав все известные сведения, проводим черту, а затем вписываем, что еще неизвестно.
  • Вопрос себе. Чтобы убедиться, что все вопросы заданы, стоит еще раз вчитаться, а затем спросить себя: «Что я ищу?»

Подбираем формулы

  1. Формулы. Выпишите все формулы, способствующие решению задачи.
  2. Преобразования. Здесь происходят сокращения, если им есть место.
  3. Уравнения. Из полученного результата составляется одно или система уравнений.

Решаем уравнения и ищем все неизвестные величины

Под получившееся уравнения нужно написать известные математические величины. Шаг повторяется под все величины под знаком «неизвестно». Стоит начать с переменных, значение которых определяется проще. Когда все неизвестные найдены, получается ответ. Он обводится прямоугольником. Готово!

Советы

  • Некоторые задачи даются к решению непросто. Множество из них требуют повышенного внимания, однако некоторые ученики не питают любви к предмету. Изучать его все же придется. Придерживаясь советов по решению уравнений из курса физики, решение задач покажется несложной процедурой, а понимать программу станет интереснее.
  • Внимательно читайте условия. Чтобы понимать способы решения, стоит несколько раз пройтись по условиям, обращая внимание на детали. Чтобы понимать, усвоились ли данные, стоит оторваться от учебника и воспроизвести ее в голове. Совпадает с написанным в учебнике? Нет? Перечитайте еще раз, представив ситуацию наглядно, словно в кино — так картинка станет реальнее!
  • Решайте для себя. Чтобы развить интерес к ходу работы, следует погрузиться в нее, понимая, что вы делаете это прежде всего для себя, а не ради оценки, репетитора, преподавателя. Так вы избавляетесь Плот того, что приходится заставлять себя возвращаться к работе.
  • Полюбите то, что делаете. Решать, чтобы решить — неправильный путь. Чтобы процесс работы был интересен, нужно полюбить физику. Как развить интерес к тому, что сложно понимать? Помните, что все неизведанное — повод к саморазвитию, а каждая новая решенная задача — новый опыт!
  • Повторения. Чтобы каждый следующий раз давался проще, желательно хотя бы раз в день решать по одной задаче. Так выработается привычка, улучшится память и восприятие условий, что в дальнейшем поможет решать новые системы уравнений в считанные минуты!
  • Задавайте вопросы. Важно задавать вопрос всякий раз, когда он возникает, не взирая на реакцию. Чем больше ответов вы получите, тем лучше будете ориентироваться в физике.
  • Берите перерывы. Иногда на задачу нужен «новый взгляд». Если ответ не поддается вычислениям уже длительное время, следует переключить свое внимание, а затем снова приступить к работе. Свежие мысли нередко моментально выдают способ решения!
  • Помните, что главное — подбор формул. Остальное — лишь подключение знаний математики. Выпишите все формулы, который на ваш взгляд могут подойти, а затем подробно разбирайте, что именно нужно в вашем случае!

Все мероприятия

23.08.2022

18:03

С каким набором предметов ЕГЭ проще всего поступить в вуз?

Расскажем о выборе предметов ЕГЭ

Читать далее

18. 08.2022

15:40

Как подготовиться к ЕГЭ по русскому языку, чтобы сдать на 80+ баллов

Расскажем о самых эффективных способах подготовки

Читать далее

05.05.2022

18:35

На какие специальности можно поступить с ЕГЭ по английскому?

Изучаем направления, доступные для сдающих иностранный язык

Читать далее

05.05.2022

14:26

Куда поступать с ЕГЭ по химии?

Сдав ЕГЭ по химии, можно поступить не только на медицинские специальности. В статье разберем все воз…

Читать далее

24.04.2022

11:01

Подготовка к ЕГЭ при вузе

Отвечаем на вопросы о подготовке к ЕГЭ в центре «Уникум» РУДН

Читать далее

23.04.2022

19:20

ТОП-5 причин выбрать курсы подготовки к ЕГЭ в центре «Уникум»

Почему школьники выбирают учебу в «Уникуме»? Расскажем в статье!

Читать далее

Как решать задачи по физике: пример решения и советы

Все мы когда-то сталкиваемся с решением задач по физике. И надо признаться, что для большинства из нас  это не самая долгожданная встреча. Тем не менее, мы знаем, что всего несколько простых шагов и нехитрых действий позволят перейти в отношениях с Физикой «на ты». Решение задач – важная составляющая процесса обучения, которую не стоит недооценивать. Ведь решение физических задач на разные темы выводит  понимание физических процессов на качественно новый уровень.

Ежедневная рассылка с полезной информацией для студентов всех направлений – на нашем телеграм-канале.

Если вы ранее никогда не сталкивались с решением задач, встает резонный вопрос: с чего начать?

Как решать задачи по физике

Чтобы решение задач по физике не было не вызывало затруднений, предлагаем следовать при решении любой задачи следующей универсальной инструкции. Совершенно не важно, нужно ли решить задачу на движение или узнать, какое количество теплоты Q выделится в ходе изобарного процесса. Данная инструкция не даст ответа на конкретную задачу, но может сделать ее решение более простым и быстрым.

  • Не спешите и не паникуйте! Помните первое правило путеводителя по Галактике: «Не паникуй». Как правило, стандартные задачи большинства курсов решаются в одно или два (ну ладно, три) действия, и ничего сверхсложного в них нет. Первым делом внимательно прочитайте условие задачи и осмыслите, что в ней требуется найти. Ознакомьтесь с похожими примерами решения задач по физике.
  • Теперь можно оформлять «ДАНО». Аккуратно выпишите все заданные величины и не забывайте о размерностях. Размерности величин целесообразно сразу перевести в систему СИ, чтобы потом не запутаться в вычислениях.
  • Очень важный пункт: РИСУНОК. Да, мы не Пикассо и не Дали, но и наших художественных способностей будет вполне достаточно. Верный поясняющий рисунок к задаче – это залог успеха и правильного решения. Визуализация данных очень хорошо помогает, и не стоит ее недооценивать. Помните, в физических задачках вечно что-то происходит — шайба летит под углом к горизонту, электрон бомбардирует пластину, идеальный газ совершает работу, отец и сын меняются местами в лодке и так далее.
    Так вот, не ленитесь и нарисуйте это! Причем не просто так, а с указанием действующих сил, векторов скоростей и прочих данных в задаче величин.
  • Теперь, когда вся картина у нас перед глазами, следует понять, на применении какого физического закона построено решение Вашей задачи. Часто это можно узнать чисто интуитивно. Если в задаче идет речь о теле, которое движется по окружности, а найти нужно момент инерции, очевидно, это задача на использование законов динамики вращательного движения. Или если дан путь и время, а найти нужно среднюю скорость – это, конечно, кинематика. Возможно, соответствующий раздел физики непосредственно перед решением задачи будет полезно проштудировать повторно.
  • Настало  время подумать, как именно найти искомую величину, зная то, что мы, собственно, знаем. Для удобства можете расположить перед глазами физические формулы. Это поможет быстрее сообразить, что откуда вытекает и как находится. Немного работы мозга — и бинго! Вы уже знаете, что делать дальше.
  • Решение целесообразно записать сначала в общем, буквенном виде. Формулу с буквами нужно привести к максимально простому виду, по возможности упростив ее. После этого можете подставлять числовые значения и переходить непосредственно к вычислениям. В конце не забудьте проверить размерность полученной физической величины. Если нужно было найти скорость, а получились килограммы, значит, где-то в решении спряталась ошибка. Будьте внимательны, и все получится!

Конечно, случается и так, что над задачей приходится попотеть. Бывают такие орешки, которые не удается расколоть с первого раза, особенно без должного опыта. Вы стараетесь изо всех сил, а решение так и не дается? Главное — никогда не сдавайтесь! Просто взгляните на Николу Тесла, и это придаст сил пробовать снова и снова!

Кстати! Для всех наших читателей сейчас действует скидка 10% на любой вид работы

 

Пример решения задачи

Маховик делал 8 оборотов в секунду. Под действием постоянного тормозящего момента 10 Н*м он остановился через 50 секунд. Определить момент инерции маховика.

Итак, начинаем решение. Найти нужно момент инерции — скалярную физическую величину, являющуюся мерой инертности тела во вращательном движении вокруг оси. Запишем дано, нарисуем маховик, и поймем, что задачу нужно решать с помощью основного уравнения динамики вращательного движения, согласно которому результирующий момент внешней силы, действующей на тело, равен произведению момента инерции тела на его угловое ускорение. Получаем решение задачи в следующем виде:

 

Надеемся, что наша универсальная и проверенная временем памятка по решению физических задач принесет пользу. Ведь лучшие авторы по физике используют ее при решении задач любой сложности. Конечно, в каждой задаче может быть изюминка, и стоит помнить, что индивидуальный подход к  задаче – важная составляющая успеха и понимания предмета. Тем не менее, все пункты, приведенные нами в списке, действительно подходят для решения любой задачи. Ну а если остались вопросы – смело задавайте их специалистам студенческого сервиса, они с радостью поделятся своими знаниями!

Автор: Иван

Иван Колобков, известный также как Джони. Маркетолог, аналитик и копирайтер компании Zaochnik. Подающий надежды молодой писатель. Питает любовь к физике, раритетным вещам и творчеству Ч. Буковски.

Задачи по физике | Репетитор по физике

Необходимость получения высокого итогового балла за ЕГЭ по физике требует от ученика определённого уровня подготовки по физике, а именно: глубокого усвоения теоретического материала, уверенного владения приёмами и методами решения задач любой сложности по всем разделам физики.

Если вам нужна помошь в обучении решению задач по физике, репетитор по физике Виктория Витальевна будет рада вам помочь. Готовлю продвинутых учеников к перечневым олимпиадам по физике, к вступительным экзаменам СУНЦ МГУ.

На ЕГЭ по физике контролируются знания и умение решать задачи из следующих разделов школьного курса физики:

  1.    Механика (кинематика, динамика, статика и гидростатика, законы сохранения в механике, механические колебания и волны) – около 35% заданий ЕГЭ
  2.    Молекулярная физика и термодинамика – около 25% заданий ЕГЭ
  3.    Электродинамика (электростатика, законы постоянного тока, электрический ток в различных средах, магнитное поле, электромагнитная индукция, электромагнитные колебания и волны) – около 25% заданий ЕГЭ
  4.    Оптика – около 5%
  5.    Квантовая физика – около 10% заданий ЕГЭ

Общее количество заданий в ЕГЭ по каждому из разделов пропорционально его содержательному наполнению  и учебному времени, отведённому на изучение данного раздела в школьном курсе физики. Качественные задачи с выбором ответов требуют глубоких теоретических знаний, умения анализировать ситуацию в необычной интерпретации.

Особое внимание следует уделить расчётным задачам высокого уровня сложности, которые проверяют знания физических явлений, формул расчёта физических величин и законов физики в изменённой или новой ситуации. Эти задачи также позволяют проверить навыки комплексного использования знаний и умений из различных разделов курса физики. Поэтому решение таких задач требует применения знаний сразу из двух-трёх разделов физики, т. е. довольно высокого уровня подготовки.

Качественная задача N 24 с подробным объяснением, опираясь на физические законы и явления, позволяет проявить глубину теоретических знаний и получить 3 балла за подробный правильный ответ.

Сложность расчётных задач определяет уровень требований для поступления на технические и физико-математические специальности большинства университетов и позволяет дифференцировать абитуриентов при их дальнейшем отборе в университеты с различными требованиями к уровню подготовки.

За каждое правильное и полное решение расчётной задачи высокой сложности с развёрнутым ответом ученик получает 3 либо 4 первичных балла, т. е. наивысший балл. Задания 25 и 26 повышенной сложности дают два первичных балла за правильное решение.

Задание 30 требует обосновать выбор того или иного физического закона при решении и даёт 4 первичных балла.

Таким образом, расчётные задачи играют значительную роль в  ЕГЭ по физике; получение высокой итоговой оценки за экзамен без решения хотя бы нескольких расчётных задач с развёрнутым ответом невозможно.

  1.   Внимательно прочитать условие задачи
  2.   Записать кратко дано и необходимые постоянные
  3.   Перевести единицы измерения в СИ
  4.   Определить раздел или разделы физики, рассматриваемые в задаче
  5.   Сделать  схематичный рисунок при решении задач на второй закон Ньютона (силы, действующие на тела, ускорение, если есть, выбрать оси координат) или законы сохранения ( нулевой уровень потенциальной энергии, начальное и конечное положения тел, скорости)
  6.   Указать законы и физические теории, используемые в решении задач и границы их применимости
  7.   Записать формулы, выражающие физические законы, зависимости, определения физических величин, применение которых необходимо для решения задач выбранным способом
  8.   Провести математические преобразования, выразив неизвестное
  9.   Произвести расчёты с указанием единиц измерения физических величин  
  10.   Проанализировать полученный результат и записать ответ

Самостоятельная подготовка к решению задач по физике очень важна на подготовительном этапе  ЕГЭ. При хорошей теоретической подготовке навык  решения задач приобретается только в процессе планомерных систематических занятий.

Не унывайте, если результаты пройденных тестов невысоки при наличнии хороших и даже отличных отметок по физике в школе. ЕГЭ по физике — это особый вид тестовой проверки и требует специальной подготовки к нему, которую можно пройти, воспользовавшись услугами опытного репетитора по физике.
Обычно, при хорошей подготовке по физике в школе, достаточно всего несколько занятий, чтобы почувствовать уверенность и быть готовым к сдаче ЕГЭ по физике.

Тренировка в решении задач поможет ориентироваться в разных типах заданий, рассчитывать время, создаст позитивный настрой на экзамене, что, безусловно, скажется на его результате.

На нашем сайте представлены задачи по физике по разделам: 

  • Кинематика
  • Динамика
  • Законы Сохранения
  • Статика и Гидростатика
  • Молекулярная физика и термодинамика
  • Электродинамика
  • Оптика
  • Квантовая физика

Решение ⚠️ задач по физике с подробным объяснением, примеры для самостоятельного изучения

Обучаясь в школе, каждый сталкивается с решением задач по физике. Не всем дисциплина дается легко.

Бытует мнение, что для успешного решения задач по этому предмету, нужно досконально разбираться в физических процессах. Это не совсем так. Мы считаем, что достаточно использовать определенный алгоритм, чтобы добиться значительных успехов. Спешим поделиться с вами ценной информацией!

Как решать задачи по физике

Итак, чтобы задачи по физике давались легко, предлагаем придерживаться следующей системы:

  1. Внимательно прочитайте условия задачи, при необходимости, несколько раз; вникнете в то, о чем говорится в тексте.
  2. Запишите условия. Все известные в задаче данные нужно записать в столбик под названием «Дано». Обратите внимание, во многих задачах по физике, данными для решения являются и названия вещества. Например, дана задача: «Сколько понадобится железнодорожных цистерн для перевозки 1000 тонн нефти, если вместимость каждой цистерны 50 \(м^3\)?». 3\). Также не забывайте про постоянные величины, например, ускорение свободного падения. В задачах на свободное падение о нем может быть не сказано ни слова, но оно предполагается в условиях и необходимо, чтобы их решить. Подумайте об этом, когда записываете все известные данные.
  3. С столбце «СИ» приведите все данные в задаче к международным единицам измерения. Так как в международной системе основными единицами измерения массы считаются килограммы (кг), массу из приведенной выше задачи необходимо привести в нужное значение: 1 000 тонн = 1 000 000 кг. 
  4. Нарисуйте схематичный рисунок. Он нужен не для всех задач. Но в тех, где упоминаются действующие на тело силы и векторы скоростей, изображение может существенно облегчить понимание процесса и натолкнуть на правильное решение.
  5. Определите неизвестную величину, ту, что необходимо узнать, решив задание. Написав в столбике все, что известно в задаче, проведите черту под известными данными и пропишите ту величину, которую будете искать.
  6. Подберите формулы. Это самый важный пункт в нашем алгоритме! Решение задачи после выбора формулы будет заключаться в математических вычислениях, которые имеют к физике лишь опосредованное отношение. На черновике выпишите те формулы, которые могут подойти для конкретной задачи и выберите ту, которая будет способствовать решению.
  7. Математические вычисления. Остальное решение задачи сводится к математике. Нужно сделать необходимые преобразования и сокращения, если они нужны. Затем составить уравнение или систему уравнений. Остается только их решить и найти все неизвестные, а в конце искомую величину. Ответ обведите в прямоугольник. 
Источник: evrophiz.wordpress.com

Примеры решения типовых задач по разделам

Рассмотрим подробнее решение задач из разных разделов физики по предложенному алгоритму. И дадим все необходимые объяснения к каждой из них.

Система абсолютно универсальна и подходит для решения заданий по динамике, кинематике, статике и другим разделам физики.

Кинематика

Источник: znanio.ru

Кинематика — это раздел механики, который изучает математическое описание движения тел. 

Данный раздел охватывает следующие темы:

  • равномерное и равноускоренное движение тел;
  • движение тела по окружности;
  • относительность движения;
  • свободное падение тел.

Рассмотрим типовые задачи на каждую из этих тем.

Равномерное и равноускоренное движение тел

Для решения задач по этой теме нужно знать уравнение движения тела, понимать, что такое средняя, постоянная скорости и ускорение, уметь выяснять их векторное направление в конкретной задаче.

Как правило, в задачах на равномерное и равноускоренное движение необходимо найти или пройденный путь (S), или скорость движения (V), или время (t).

Задача:

Поезд длиной 240 метров, двигаясь равномерно, прошел мост за 2 минуты. Какова была скорость поезда, если длина моста равна 360 метрам?

Решение:

  1. Записываем известные нам данные:\( l_1=240\) м. , \(l_2=360\) м., \(t=2\) мин., \(V\)=?
  2. Проводим необходимые преобразования времени до принятых в мире единиц измерения — секунд: 2 минуты = 120 секунд.
  3. Мы знаем, что скорость равномерного движения определяется по формуле: \(V=\frac st\) 
  4. Время нам известно, для того, чтобы найти скорость, нужно сначала определить путь пройденный поездом. Если мы схематично изобразим перемещение поезда по мосту, то увидим, что путь, пройденный поездом, равен длине самого поезда плюс длине самого моста, т.е. \(s=l_1+l_2\).
  5. Переходим к математическим вычислениям: \(s=240+360=600\) метров.
  6. \(V=600/120= 5\) м/с.

Задача:

При равноускоренном движении с начальной скоростью 5 м/с тело за 3 секунды прошло 20 метров. С каким ускорением двигалось тело? Какова его скорость в конце третьей секунды?

Решение:

  1. Фиксируем данные известных нам величин: \(V_1=5 \) м/с, \(t=3\) с, s=20 м. 2\)
Свободное падение тел

Для решения задач по этой теме нужно знать закон движения при свободном падении и закономерность изменения скорости тела со временем, а также помнить про постоянную величину — коэффициент силы тяжести.

В задачах на свободное падение тел может быть предложено найти скорость движения тела, высоту, с которой оно падало или время его движения.

Задача:

Камень брошен вниз с высоты \(85\) метров. Он летит со скоростью \(8\) м/с. С какой скоростью он ударяется о землю?

Решение:

  1. Определяем известные и неизвестные нам данные: \(h=85\) метров, \(V_1=8\) м/с., \(V_2=?\) Мы помним, что на любое падающее тело воздействует коэффициент силы тяжести, равный \(9,8\) Н/кг.
  2. У нас есть все вводные для определения конечной скорости по формуле: \(V_2=V_1+g\times t\)
  3. Подставляем числовые значения в уравнение и получаем скорость тела в момент удара о землю, равную \(41,3\) м/с.
Относительность движения

Задачи на относительность движения всегда требует выбрать неподвижную систему координат, относительно которой и будут производиться все расчеты. В таких заданиях ученикам обычно предлагают найти относительную скорость объекта, минимальное время, продолжительность пути или длину объекта.

Задача:

Два поезда движутся навстречу друг другу по параллельным ж/д путям. Один — со скоростью 72 км/ч, другой — со скоростью 54 км/ч. Пассажир первого поезда отмечает, что второй проходит мимо него в течение 10 секунд. Определите длину второго поезда.

Решение:

  1. Записываем известные нам данные: \(V_1=72\) км/ч, \(V_2=54\) км/ч, \(t=10\) с, \(l_2=?\)
  2. Переводим км/ч в м/с: \(V_1= 20\) м/с, \(V_2=15\) м/с.
  3. Определяем систему координат, от которой будем отталкиваться при вычислении искомой величины. Логично будет, если такой системой станет линейная система координат, связанная с первым поездом и направленная по ходу его движения. Получается, что второй поезд двигается со скоростью \(V_2=15\) м/с в направлении со скоростью \(V_1=20\) м/с.
  4. Находим общую скорость движения по формуле: \(V=V_1+V_2\) 
  5. Она равна \(35\) м/с.
  6. Определяем длину поезда по формуле: \(l_2=V\times t\)
  7. Получаем длину поезда, равную \(350\) метрам.

Динамика

Источник: infourok.ru

Динамика — это раздел физической дисциплины, который изучает взаимодействие тел друг другом, причины изменения движения тел и силы, воздействующие на тело в тот или иной момент времени.

Этот раздел механики охватывает следующие темы:

  • законы Ньютона; 
  • неподвижный блок и наклонная поверхность; 
  • закон всемирного тяготения;
  • сила упругости, упругий и неупругий удар; 
  • работа, энергия, мощность;
  • закон сохранения энергии и импульса.

Для выполнения заданий по динамике необходимо знать законы Исаака Ньютона, силы, воздействующие на тела, закон сохранения импульса и уметь рисовать несложные рисунки, иллюстрирующие движение и взаимодействие тел. 2}\) вычисляем силу притяжения между книгами.

  • Произведя математические вычисления получаем ответ: книги притягиваются друг к другу с силой приблизительно равной \(2,4\) Н.
  • Сила упругости

    Задача: 

    К покоящейся на горизонтальной поверхности системе, которая состоит из куба массой 1 кг и 2-х пружин, приложена постоянная горизонтальная сила величиной 25 Ньютонов. Между кубом и поверхностью трения нет. Жесткость первой пружины составляет  \(450 Н/м\), жесткость второй пружины \(550 Н/м\). Определите удлинение пружин.

    Решение:

    1. Записываем в столбце «Дано» данные, которые нам известны: \(m=1\) кг,\( F=25\) Н, \(k_1=450\) Н/м, \(k_2=550\) Н/м, \(\Delta l_1=?\), \(\Delta l_2=? \) 
    2. Согласно 3-му закону Ньютона \(F=F_упр\)
    3. По закону Гука \(F_упр=F=k\times\Delta l\) отсюда выводим формулы для нахождения удлинения пружин: \(\Delta l_1=\frac F{k_1}\) и \(\Delta l_2=\frac F{k_2}\)
    4. Подставляем известные нам числовые значения в формулы и получаем ответ: \(6 см\) — удлинение первой пружины, \(5 см\) — удлинение второй пружины. 2.\)
    5. Переводим минуты в часы: \(1\) минута=\(60\) секунд.
    6. Найти работу можно по формуле: \(A=F\times S\)
    7. В данных условиях \(S=h\), а \(F=g\times m\)
    8. В условиях задачи нет значения массы тела, но мы помним, что массу можно найти по формуле: \(m=p\times V\)
    9. Формула нахождения работы приобретает следующий вид: \(A=p\times V\times g\times h\)
    10. Подставляем известные числовые значения в формулу и получаем ответ: работа = 3 528 000 000 Дж = 3 528 МДж. 
    Закон сохранения энергии и импульса

    Задача:

    Тепловоз массой 130 тонн приближается со скоростью 2 м/с к неподвижному составу массой 1170 тонн. С какой скоростью будет двигаться состав после сцепления с тепловозом? 

    Решение:

    1. Записываем известные нам данные: \(m_1=130\) тонн, \(V_1=2\) м/с, \(m_2=1170\) тонн,  \(V_2=0\) м/с, V=?
    2. Согласно закону сохранения импульса \(m_1\times V_1+m_2\times V_2=m_3\times V_3\)
    3. Из этой формулы получаем уравнение для нахождения скорости состава после сцепления: \(V_3=\frac{m_1\times V_1}{m_1+m_2}\)
    4. Подставляем известные нам значения в формулу и получаем искомую скорость, равную \(0,2\) м/с.

    Статика

    Источник: infourok.ru

    Статика — третий раздел механики, который изучает механические системы в условиях равновесия и действие приложенных к ним сил.

    Для решения задач по статике необходимо обязательно рисовать схемы, иллюстрирующие заданные процессы, определять модули и направления сил, пользоваться законами сопротивления материалов.

    Статика включает в себя следующие разделы:

    • равновесие тел;
    • давление в жидкостях и газе;
    • закон Архимеда.
    Равновесие тел
    Источник: infourok.ru
    Давление в жидкостях и газе

    Задача:

    Водолаз в жестком скафандре может погружаться на глубину 250 метров, искусный ныряльщик — на 20 метров. Определите давление воды в море на этих глубинах.

    Решение:

    1. Записываем известные нам данные из условия задачи: \(h_1\)=250 м, \(h_2\) =20 м, \(p=1030 кг/м^3\), \(g=9,8\) Н/кг, \(p_1=?,\) \(p_2=?\)
    2. По формуле \(P_1=p\times g\times h_1\) определяем давление воды для водолаза, оно будет равно примерно 2524 кПа. 3.\)

    Молекулярная физика

    Молекулярная физика — это один из разделов физики, описывающий физические свойства объектов путем изучения их молекулярного строения.

    В основе всех задач по молекулярной физике лежит уравнение молекулярно-кинетической теории: \(P=\frac13\times m_0\times n\times V_2\)

    Источник: znakka4estva.ru

    Термодинамика

    Источник: present5.com

    Термодинамика — физический раздел, который изучает общие свойства макроскопических систем, способы передачи и превращения энергии в них.

    В раздел термодинамики входят следующие темы:

    • теплота сгорания топлива; 
    • изменение внутренней энергии тела при совершении работы; 
    • внутренняя энергия идеального газа;
    • первый закон термодинамики;
    • КПД теплового двигателя.
    Теплота сгорания топлива

    При решении задач на сгорание топлива, важно помнить про удельную теплоту сгорания каждого вида топлива. 7\) Дж/кг, \(Q=?\)

  • По формуле \(Q=q\times m\) определяем теплоту сгорания и получаем 95 кДж.
  • Изменение внутренней энергии тела при совершении работы

    Задача:

    Вычислите внутреннюю энергию 1 килограмма воды при ее нагревании на 2 Кельвина.

    Решение:

    1. Записываем известные и неизвестные величины из условий задачи: \(m=1\)  кг, \(T=2\)К, \(U=?\), не забываем про удельную теплоемкость воды \(c=4200\) Дж/кгхК.
    2. Количество теплоты, которое получит вода, будет затрачено на изменение ее внутренней энергии, т.е. \(U=Q\).
    3. \(Q=c\times m\times T\) следовательно, \(U=c\times m\times T\)
    4. Подставляем числовые значения в формулу и получаем ответ: 8400 Дж.
    Внутренняя энергия идеального газа, первый закон термодинамики

    При решении таких задач важно помнить про молярную массу вещества и универсальную газовую постоянную.

    Задача:

    Чему будет равна внутренняя энергия гелия массой 200 грамм при условии, что температура будет увеличена на 20 Кельвинов? 

    Решение:

    1. Фиксируем известные величины: \(m=200\) г, \(\Delta T= 20\) К. 3\).
    2. Коэффициент полезного действия определяется по формуле: \(\eta=\frac{A_п}{A_з}\)
    3. \(A_п\) равна количеству теплоты (\(Q\)), которое необходимо для изменения температуры воды. \(A_п=Q=c\times m\times T.\) Массу воды найдем по формуле: \(m_1=p_1\times V_1\)
    4. \(A_з\) равна количеству теплоты, выделенному при сгорании керосина массой 80 грамм, следовательно, \(A_з=q\times m_2\)
    5. Подставив все известные величины в формулу, получаем ответ: КПД = 0,33.

    Электростатика

    Источник: infourok.ru

    Электростатика — это раздел физики об электричестве, который изучает взаимодействие электрических зарядов, находящихся в неподвижности.

    К задачам по электростатике относятся задачи на :

    • закон Кулона; 
    • напряженность и работу электростатического поля; 
    • электроемкость.
    Закон Кулона

    Задача:

    Определите силу взаимодействия двух одинаковых точечных зарядов по 1 микро кулону, которые находятся на расстоянии 30 сантиметров друг от друга. 2/м\).

  • Согласно закону Ома \(I=\frac UR\) отсюда \(U=U=I\times R\)
  • Сопротивление определяем по формуле: \(R=p\times\frac lS\)
  • Подставляем числовые данные, находим сопротивление. Оно равно 0,34 Ом.
  • Находим значение напряжения: 1,7 В.
  • Задача на работу и мощность тока:

    Определите мощность и работу электродвигателя вентилятора за 10 минут, если при напряжении 220 Вольт сила тока в электродвигателе составила 1 Ампер.

    Решение:

    1. Записываем условия: \(t=10\) мин, \(U= 220\) В, \(I=1 А\), \(P=?\) \(A=?\)
    2. Переводим минуты в секунды, получаем 600 секунд.
    3. По формуле \(P=I\times U\) определяем мощность тока. Она равна 220 Вт.
    4. По формуле \(A=P\times t\) находим работу, получаем 132000 Дж или 132 кДж.
    Магнитное поле

    К задачам раздела «Магнитное поле» относятся задания на:

    • силу Ампера;
    • силу Лоренца; 
    • магнитный момент, индукцию и самоиндукцию, энергию магнитного поля. -15 Н\).
    • Задача на магнитный поток и ЭДС индукции:

      Источник: kopilkaurokov.ruИсточник: kopilkaurokov.ru

      Колебания и волны

      Источник: prezentacii.org

      В разделе физики «Колебания и волны» изучают следующие темы:

      • механические гармонические колебания математических маятников;
      • пружинный маятник; 
      • энергия механических колебаний; 
      • механические волны; 
      • колебательный контур;
      • электромагнитные волны.

      Задача на колебания математического маятника:

      Источник: videouroki.net

      Задача на пружинный маятник:

      Источник: znanio.ru

      Задача на колебательный контур:

      Источник: mypresentation.ru

      Для того, чтобы задания по физике решались совсем легко, предмет нужно полюбить. Если это не про вас, не переживайте! Посвящайте свое время любимым дисциплинам и хобби, а физику оставьте для профессионалов Феникс. Хелп.

      Узнаем как научиться решать задачи по физике: полезные советы педагогов

      Вопрос о том, как научиться решать задачи по физике, волнует большинство школьников. Эта наука дается тяжело даже самым умным детям, поскольку она содержит много теории, которую необходимо уметь применять на практике. Задачи – это способ обучения, который учителя используют для того, чтобы дети освоили предмет с практической точки зрения, поняли, для чего нужна физика и как ее можно использовать в повседневной жизни.

      Книга «Как научиться решать задачи по физике, 7 класс»

      Поскольку физика – это наука, которую нужно осваивать постепенно, переходя от простого материала к сложному, вникать в азы предмета необходимо с первого школьного урока. Обычно впервые ученики сталкиваются с этим предметом в 7 классе. Поскольку изучение физики – это давний и наболевший вопрос для школьников, на сегодня разработано множество учебных пособий, которые значительно облегчают процесс решения задач.

      Одним из успешных авторов, которые пользуются спросом среди школьников и их родителей, является Л. Орловская. Как научиться решать задачи по физике, она подробно описывает в своей книге для учеников 7 класса. Именно в этом возрасте у детей формируется впечатление о науке. Если они с самого начала смогут положительно к ней относиться, то проблем с пониманием предмета не будет и в дальнейшем.

      Книгу Орловской можно использовать и как учебное пособие, и как справочник по физике. Кроме того, учебник рассчитан не только на аудиторию школьников. Полезную информацию в нем найдут также и родители, и учителя.

      Общие рекомендации педагогов

      Как правило, многие современные школьники пренебрегают советами учителя, стараясь найти особенную методику решения задач. И в этом состоит их самая большая ошибка. Рекомендации преподавателя действовали во все времена, если школьники относились к ним серьезно.

      Вот такие советы обычно дают учителя:

      • Внимательно читайте условие задачи. Профессиональные педагоги уверены, что если полностью разобраться в условии, то задание автоматически будет решено наполовину.
      • Рисуйте схемы для наглядности. Практически к любой задаче по физике можно нарисовать график, рисунок или чертеж. Это поможет вам осознать смысл решения.
      • Расписывайте решение в мельчайших подробностях. Так вы увидите наиболее полную картину, сможете устранить недостатки и проверить себя в случае необходимости.

      Если вы не знаете, как научиться решать задачи по физике, то попробуйте следовать этим советам неукоснительно. Скорее всего, вы очень быстро заметите, что объем ваших знаний значительно увеличится.

      Психологическая подготовка к занятию

      Многие школьники недооценивают роль правильного психологического настроя при решении задач. На самом же деле он лежит в основе учебного процесса. При правильном настрое вы не только сможете спокойно преодолеть все трудности, но и принять свой успех как должное.

      Итак, воспользуйтесь алгоритмом для создания нужной мотивации:

      • Успокойтесь и поймите, что перед вами всего лишь задача. Ничего не случится, если с первого раза вы ее не решите.
      • Изучите условие задачи, постарайтесь осознать его смысл.
      • Нарисуйте схему к задаче, даже если это не задано по условию. Это значительно упростит процесс решения.
      • Составьте краткое условие задачи, в котором будет присутствовать только нужная вам информация.
      • Сформулируйте вопрос, на который вам нужно ответить в письменном виде.
      • Посмотрите на сформировавшуюся картину и осознайте, что половина решения у вас готова.

      Эти простые шаги не только приведут вас к верному решению, но и помогут сформировать уверенность в себе. Как только вы поймете, что ничего сложного вас не ожидает, а вы являетесь вполне способным человеком, приступайте непосредственно к решению.

      Алгоритм решения задачи

      Когда вы поняли, с какими числами и какой информацией вам придется работать, осознали суть и смысл задания, можно приступать к решению. Его алгоритм выглядит так:

      • Выпишите для наглядности все формулы, которые могут быть вам полезны. Пусть они всегда будут перед глазами.
      • Проанализировав все формулы, выберите только нужные, вычеркнув остальные.
      • Подставьте числа в формулы, решив примеры. Если у вас получились уравнения, то найдите неизвестную переменную. Тут вам помогут знания математики.
      • Если задание объемное, то повторяйте предыдущее действие, пока не найдете все неизвестные значения.
      • После описания решения сформулируйте конечный ответ.

      Людям, которые ходят разобраться, как научиться решать олимпиадные задачи по физике, этот алгоритм тоже подходит. Просто некоторые его пункты придется повторить многократно.

      Полезные советы

      Если какая-либо наука и нуждается в дополнительных советах для выполнения практических заданий, то это физика. Задача, которая легко решается, скорее всего, просто неправильно вами понята. Или же вы настолько разобрались в этой науке, что в обучении больше не нуждаетесь. Из этого вытекает первый совет. Он заключается в том, что вам нужно постоянно практиковаться. Чем большее количество задач вы решите, тем быстрее выработаете автоматизм. Другие рекомендации профессиональных педагогов:

      • Вся изучаемая информация основана на теории, причем самой простой. Она изучается в самом начале курса физики. Поэтому не пренебрегайте учебниками для 7 класса, если какие-то сведения были вами забыты.
      • Если вы долгое время не можете найти решение, сделайте перерыв на несколько часов, а после этого снова приступайте к раздумьям.
      • Если вы так и не поняли, как научиться решать задачи по физике, постарайтесь изучить всю теорию. Скорее всего, у вас недостаточная база знаний.
      • Не стесняйтесь попросить о помощи.
      • Все задачи по физике основаны на понимании их смысла. Поэтому не пытайтесь просто совершать математические действия, которые вам не ясны.

      Изучите эти советы, чтобы на следующем занятии по физике применить их практически.

      Особое предупреждение

      Иногда случается, что человек не может понять физику, потому что она ему не дается. Это обосновано гуманитарным складом ума. Не расстраивайтесь, если вы относитесь к этой категории. Понимающий преподаватель поможет вам освоить азы науки, которые вам понадобятся для получения достойной оценки.

      ЕГЭ по физике с решениями, часть А

      Задачи ЕГЭ по физике с готовыми решениями хорошо подходят для подготовки. Часть А.

      Решение задач по физике, ЕГЭ – часть С

      A1

      Зависимость координаты от времени для некоторого тела описывается уравнением . В какой момент времени проекция скорости тела на ось равна нулю?

      Решение: По виду уравнения зависимости координаты от времени заключаем, что движение равноускоренное с отрицательной проекцией ускорения. Уравнение зависимости скорости от времени имеет вид: . Определяем значения начальной скорости v0=12 м/с и ускорения, равного удвоенному коэффициенту при t2 (а=4 м/с2). Следовательно, уравнение скорости в нашем случае имеет вид: . Подставляя v=0, находим t=3с.

      Верный ответ 2

      1) 6с

      2) 3 с

      3) 2с

      4) 0

      A2

      Тело движется вдоль оси Ох под действием силы F. Проекция скорости тела меняется по закону, представленному на рисунке. По какому закону изменяется проекция силы Fх?

      1                               2                              3                  4

      Решение: Из анализа графика следует, что движение тела равноускоренное с отрицательной проекцией ускорения. Такое движение осуществляется под действием постоянной по модулю силы, проекция которой на направление движения отрицательна.

      Верный ответ 3

      1) 1

      2) 2

      3) 3

      4) 4

      A3

      В каком случае потребуется большая сила, чтобы сдвинуть верхний брусок с места? Материал, из которых сделаны бруски, а также их массы одинаковы.

      1) в первом

      2) во втором

      3) в третьем

      4) во всех случаях сила одинакова

      Решение: Поскольку максимальная сила трения покоя примерно равна силе трения скольжения, то для того, чтобы сдвинуть брусок с места необходимо приложить силу по величине равную Fтр=μN, где N- сила нормальной реакции опоры. Поскольку тело находится на горизонтальной поверхности, N= mg. следовательно, Fтр=μ mg. Поскольку все бруски имеют одинаковую массу, то и сила, необходимая для т ого, чтобы сдвинуть их с места, должна быть одинаковой.

      Верный ответ 4

      A4

      Шарик массой m, двигаясь со скоростью V перпендикулярно стенке, упруго отскакивает от нее в обратную сторону с прежней по модулю скоростью. Чему равен модуль импульса силы, действовавшей на шарик в момент удара?

      Решение: Модуль импульса силы, действовавшей на шарик в момент удара, равняется модулю изменения импульса шарика |Dp|=2mv.

      Верный ответ 3

      1) 0

      2) mV

      3) 2mV

      4) mV/2

      A5

      Машина равномерно поднимает тело массой 20 кг на высоту h=10 м за время t=20 с. Чему равна ее мощность?

      Решение: Поскольку тело движется равномерно, работа силы тяги по модулю равна работе силы тяжести. А= mgh. Тогда мощность определится следующим образом: . После подстановки и вычислений получим N=100 Вт.

      Верный ответ 1

      1) 100 Вт

      2) 10 Вт

      3) 1000 Вт

      4) 1 Вт

      A6

      На рисунке изображена поперечная волна. Частота колебаний частиц среды, в которой она распространяется, 4 Гц. Чему равна скорость волны?

      Решение: Скорость волны равна произведению ее длины волны на частоту колебаний частиц среды. Из рисунка видно, что половина длины волны равна 8 см, следовательно, длина волны 0,16 м. Умножая полученное значение на частоту (4 Гц), получим значение скорости, равное 0,64 м/с.

      Верный ответ 1

      1) 0,64 м/с

      2) 0,32 м/с

      3) 32 м/с

      4) 64 м/с

      A7

      На столе лежит книга массой 0,5 кг. Какая из указанных ниже сил, согласно третьему закону Ньютона, равна по модулю и противоположна по направлению силе тяжести, действующей на книгу?

      Решение: Сила тяжести обусловлена взаимодействием книги с Землей. По третьему закону Ньютона силой, равной по модулю и противоположной по направлению действующей на книгу силе тяжести,  является сила тяготения, действующая на Землю со стороны книги.

      Верный ответ 3

      1) сила реакции опоры

      2) вес книги

      3) сила тяготения, действующая на Землю со стороны книги

      4) сила трения покоя

      A8

      Укажите пару веществ, скорость диффузии которых наибольшая при прочих равных условиях:

      Решение: Наибольшая скорость диффузии при прочих равных условиях наблюдается в газах.

      Верный ответ 2

      1) раствор медного купороса и вода

      2) пары эфира и воздух

      3) свинцовая и медная пластины

      4) вода и спирт

      A9

      Медь плавится при постоянной температуре 1085° C. Поглощается или выделяется энергия в этом процессе?

      Решение: Плавление меди происходит с поглощением энергии, поскольку внутренняя энергия расплава больше внутренней энергии меди в твердом состоянии.

      Верный ответ 1

      1) поглощается

      2) выделяется

      3) не поглощается и не выделяется

      4) может поглощаться, может выделяться

      A10

      2 моль неона и 3 моль аргона находятся в разных сосудах при одинаковой температуре. Отношение значений внутренних энергий этих газовравно

      Решение: Внутренняя энергия неона и аргона определяется следующими с отношениями: , . Поскольку значения всех величин, входящих в правые части этих равенств, за исключением n, одинаковы, отношение значений внутренних энергий определяется отношением .

      Верный ответ 3

      1) 3/2

      2) 4/3

      3) 2/3

      4) 1/3

      A11

      В алюминиевый сосуд массой 100 г налито 200 г воды. Температура воды и стакана 750С. При опускании в воду серебряной ложки массой 80 г при температуре 150С температура воды в сосуде понизится на

      Решение: В теплообмене участвуют три тела: вода, алюминиевый стакан и серебряная ложка. При этом изменения агрегатного состояния не происходит. Уравнение теплового баланса имеет вид:, где mв, mст и mл – массы воды, стакана и ложки соответственно, св, сал и сс – удельные теплоемкости воды, алюминия и серебра, t1– начальная температура воды и стакана, t2 – начальная температура ложки, q – температура термодинамического равновесия. Из уравнения находим q = 73,80С. Следовательно температура воды в сосуде понизится на 1,20С.

      Верный ответ 4

      1) 20С

      2) 1,50С

      3) 10С

      4) 1,20С

      A12

      Идеальный одноатомный газ находится в сосуде с жесткими стенками объемом 0,5 м3. При нагревании его давление возросло на 4∙103 Па. При этом внутренняя энергия газа увеличилась на

      Решение: Записывая уравнение Менделеева – Клапейрона (1) для начального и конечного состояний и вычитая из второго уравнения первое, получим (2). Изменение внутренней энергии идеального одноатомного газа (3) или, с учетом (2), . Подставляя числовые значения, получимкДж.

      Верный ответ 2

      1) 2 кДж

      2) 3 кДж

      3) 1,5 кДж

      4) 3 Дж

      https://5-ege.ru/ege-po-fizike-s-resheniyami-chast-a/

      A13

      Расстояние между обкладками конденсатора уменьшили в 4 раза, не отключая его от источника зарядов. При этом напряжение на обкладках конденсатора

      Решение: Изменение расстояния между обкладками конденсатора без отключения его от источника зарядов приводит к изменению его емкости и заряда на обкладках конденсатора, напряжение при этом не меняется.

      Верный ответ 4

      1) уменьшилось в 4 раза

      2) увеличилось в 4 раза

      3) увеличилось в два раза

      4) не изменилось

      A14

      На рисунке представлен участок электрической цепи. Каково отношение количеств теплоты , выделившихся на резисторах R2 и R3 за одно и то же время?

      Решение: (1), где I2 и I3 – токи, которые текут на верхнем и нижнем участке цепи. Поскольку напряжение на параллельно соединенных участках одинаково, I2*(R1+R2)= I3*(R3+R4), а . Подставляя числовые значения в формулу (1), получим

      Верный ответ 3

      1) 0,44

      2) 0,67

      3) 0,9

      4) 1,5

      A15

      При увеличении в 2 раза индукции однородного магнитного поля и площади неподвижной рамки поток вектора магнитной индукции

      Решение: Магнитный поток определяется следующим образом: Ф= B*S*cosa Следовательно, при увеличении в 2 раза индукции однородного магнитного поля и площади неподвижной рамки поток вектора магнитной индукции увеличится в 4 раза.

      Верный ответ 3

      1) не изменится

      2) увеличится в 2 раза

      3) увеличится в 4 раза

      4) уменьшится в 4 раза

      A16

      При прохождении электромагнитных волн в воздухе происходят колебания

      Решение: При прохождении электромагнитных волн в воздухе происходят колебания

      напряженности электрического и индукции магнитного полей

      Верный ответ 3

      1) молекул воздуха

      2) плотности воздуха

      3) напряженности электрического и индукции магнитного полей

      4) концентрации кислорода

      A17

      Дано: преломление светового пучка на границе стекло-воздух. Угол падения равен 60 градусов, а угол преломления – 30. Чему равен показатель преломления стекла?

      Решение: Показатель преломления , где угол падения a=60о, а угол преломления g=30о. Подставляя значения синусов в формулу (1), получим n=

      Верный ответ 3

      1) 1

      2)

      3)

      4)

      A18

      При прохождении света через стекло наибольшая скорость у лучей

      Решение: оранжевого цвета.

      Верный ответ 1

      1) оранжевого цвета

      2) синего цвета

      3) зеленого цвета

      4) голубого цвета

      A19

      Два точечных электрических заряда q1=4 мкКл и q2=10 мкКл находятся на расстоянии r друг от друга. Каким образом нужно перераспределить заряды, чтобы сила взаимодействия между ними была наибольшей?

      Решение: По закону Кулона сила взаимодействия двух точечных зарядов, находящихся на определенном неизменном расстоянии, прямо пропорциональна их произведению. При неизменном значении суммарного заряда наибольшее значение силы Кулона получается в случае равных зарядов. Наиболее просто в этом случае ответ может быть получен выбором произведения величин зарядов, приведенных в вариантах возможных ответов.

      Верный ответ 3

      1) q1=1 мкКл; q2=13 мкКл

      2) q1=6 мкКл; q2=8 мкКл

      3) q1=q2=7 мкКл

      4) q1=14 мкКл; q2=0 мкКл

      A20

      На рисунке представлена диаграмма энергетических уровней атома. Какой цифрой обозначен переход, соответствующий в спектре испускания атома самой большой частоте излучения?

      Решение: Поскольку энергия фотона пропорциональна частоте излучения, самая большая частота излучения соответствует переходу, обозначенному на рисунке цифрой 3.

      Верный ответ 3

      1) 1

      2) 2

      3) 3

      4) 4

      A21

      На рисунке приведена зависимость числа нераспавшихся ядер N от времени в процессе радиоактивного распада для трех изотопов. Для какого из них период полураспада минимален?

      Решение: Определим по графику период полураспада Т для каждого изотопа. Для этого нанесем вспомогательные линии (показаны штрихом) Видно, что изотоп II обладает наименьшим периодом полураспада.

      Верный ответ 2

      1) 1

      2) 2

      3) 3

      4) у всех изотопов одинаков

      A22

      Для возникновения цепной реакции при делении тяжелых ядер наиболее существенно соотношение числа образующихся в ядерной реакции и поглощаемых в системе

      Решение: нейтронов.

      Верный ответ 2

      1) γ-квантов

      2) нейтронов

      3) α-частиц

      4) электронов

      A23

      В опытах по фотоэффекту взяли пластину из металла с работой выхода 3,4∙10–19 Дж и стали освещать ее светом частоты 3∙1014 Гц. Затем частоту увеличили в 2 раза, оставив неизменным число фотонов, падающих на пластину за 1 с. В результате этого число фотоэлектронов, покидающих пластину за 1 с,

      Решение: Фотоэффект наблюдается в том случае, если энергия фотона hn больше работы выхода. Подстановка числовых значений и расчет энергии фотона показывает, что при освещении пластины светом частоты 3∙1014 Гц энергия фотона меньше работы выхода и фотоэффект не наблюдается. При увеличении частоты в 2 раза энергия фотона увеличивается тоже в 2 раза и становится достаточной для того, чтобы фотоэффект имел место.

      Верный ответ 2

      1) не изменилось

      2) стало не равным нулю

      3) увеличилось в 2 раза

      4) увеличилось менее чем в 2 раза

      A24

      Какова температура кипения воды при нормальной атмосферном давлении по абсолютной шкале температур?

      Решение: Температура кипения воды при нормальной атмосферном давлении t =100оС. Используя формулу, связывающую температурные шкалы Кельвина и Цельсия T=t+273, получим Т=373 К

      Верный ответ 4

      1) 100 К

      2) 173 К

      3) 273 К

      4) 373 К

      A25

      Космонавты исследовали зависимость силы тяжести от массы тела на посещенной ими планете. Погрешность измерения силы тяжести равна 4 Н, а массы тела – 50 г. Результаты измерений с учетом их погрешности представлены на рисунке. Согласно этим измерениям, ускорение свободного падения на планете приблизительно равно

      Решение: Зависимость силы тяжести от массы определяется формулой F=mg. Через начало координат и экспериментальные точки на координатной плоскости проведем прямую (показана штрихом). Ускорение свободного падения равняется тангенсу угла наклона прямой к оси m. Отсюда следует, что определенное с погрешностью значение ускорения свободного падения g=7,5 м/с2. Приведенное в ответах наиболее близкое значение 7 м/с2.

      Верный ответ 2

      1) 10 м/с2

      2) 7 м/с2

      3) 5 м/с2

      4) 2,5 м/с2

      Рекомендуем:

      Обратите внимание: на главной странице есть множество материалов для подготовки к ЕГЭ по русскому языку и другим предметам.

      Как решить задачу по физике | Джозеф Меллор

      Чтобы стать мастером физики, нужно много работать, но изучение методов и рекомендаций, изложенных в этой статье, — это первый шаг.

      Автор сделал все изображения в этой статье, используя LaTeX, tikz, numpy, pyplot и GIMP.

      Люди часто говорят о физике, как если бы она была просто набором фактов и уравнений: F=ma , энергия не может быть ни создана, ни уничтожена, E = mc² , звезды представляют собой гигантские шары водорода, сливающиеся в гелий и более тяжелые элементы и т. д. Эти факты и уравнения могут либо составлять основу нашего понимания, либо являться результатом нашего понимания, но заниматься физикой значит соединять их. Другими словами, для изучения физики требуется методы обучения и рекомендации наряду с фактами и уравнениями.

      Я не смогу подробно описать все методы и рекомендации, используемые в каждой задаче, но я хочу рассказать о некоторых из наиболее распространенных методов и рекомендаций, используемых в физике.

      За исключением особого случая, когда вас просят вывести уравнение, вы можете использовать данные уравнения для решения приличного количества задач, используя некоторые простые упражнения и базовую алгебру. Если вы знаете все значения в уравнении, кроме одного, вы можете получить это значение .

      Пример: Небесная механика

      Сколько времени понадобилось бы Земле, чтобы совершить оборот вокруг Солнца, если бы масса Солнца составляла одну сотую нынешней, а орбита Земли не изменилась?

      Третий закон Кеплера связывает массу Солнца, радиус/большую полуось орбиты и период орбиты, поэтому мы можем получить период, решив третий закон Кеплера для T и подключив в уже известных нам значениях.

      Продвинутый трюк

      Всякий раз, когда у вас есть задача по физике, которая изменяет несколько значений (в данном случае массу солнца) в формуле, попробуйте выразить свой ответ в терминах значений до изменения. Для этой задачи нам не нужно искать какие-либо из этих физических констант в задаче, и нам даже не нужен калькулятор.

      Обратите внимание, что сине-зеленые выражения одинаковы, поэтому мы можем подставить их обратно, не зная ничего, кроме того, что Земле требуется год, чтобы совершить оборот вокруг Солнца. Вы можете получить аналогичный результат, разделив период Земли на новый период и сократив члены.

      Расширение этого принципа

      Я рассмотрел только случай, когда у вас есть сингулярное уравнение с одним неизвестным, но если у вас есть n уравнений, вы можете иметь n неизвестных значений . Позже мы рассмотрим конкретные примеры.

      Если вы когда-нибудь окажетесь в ситуации, когда вы не знаете, как перейти от информации, данной в задаче, к тому, что задача требует от вас, возьмите информацию, которая у вас есть, и сделайте что-нибудь с ней, даже если вы не не знаю, чем это может вам помочь . Физика часто требует от вас взять то, что вы знаете, найти уравнение, которое может дать вам больше информации, а затем повторять процесс, пока у вас не будет достаточно информации, чтобы ответить на вопрос. Следуя этой технике, не забывайте просматривать всю имеющуюся у вас информацию, даже если вы уже использовали ее раньше.

      Пример: кинематика

      Какой длины должна быть взлетно-посадочная полоса для самолета массой м движущегося с постоянной результирующей силой F для достижения скорости vf из состояния покоя?

      Фраза «Какой длины должна быть взлетно-посадочная полоса» означает, что вы ищете расстояние. Вы знаете массу, силу, начальную скорость (0 м/с, так как вы находитесь в состоянии покоя) и конечную скорость. Поскольку вы пытаетесь использовать явное уравнение, если это возможно, вы ищете любое уравнение в своем листе уравнений, но не находите ни одного. Помня, что вы пытаетесь получить всю возможную информацию, вы вместо этого ищете уравнения, в которых вам известны все переменные, кроме одной. Вы найдете классический F = MA и осознайте, что вы знаете F и M , так что теперь у вас есть A , Acceleration. Теперь вы ищете уравнения, в которых известны все переменные, кроме одной, и обнаруживаете, что одно из кинематических уравнений имеет a , s (расстояние/перемещение), vi (начальная скорость) , и vf (конечная скорость):

      Понимая, что вы знаете все, кроме расстояния, вы можете найти расстояние и решить задачу.

      С большим опытом

      По мере накопления опыта вы будете помнить такие схемы, как «использование F = ma , а затем использование кинематического уравнения для получения скорости, времени или расстояния» и вам не придется просматривать все уравнения на листе уравнений, чтобы найти соответствующее уравнение. За все время, которое вы сэкономили, вы можете работать над другими проблемами.

      Я дам вам методы, которые работают в примерах для последующих разделов, но я понял их только потому, что продолжал следовать этому правилу, делать ошибки и учиться на них. Есть очень много способов сделать ошибку, решая проблему. Пока вы продолжаете пытаться и стараетесь не совершать ошибок, которые вы уже совершили , у вас кончатся ошибки, которые нужно совершать.

      В декартовых координатах (т. е. в стандартной системе координат (x, y, z) ) каждое измерение является ортогональным, то есть вы можете рассматривать каждое измерение по отдельности. С практической точки зрения, вы можете создать систему уравнений с одним уравнением на измерение. Вы можете использовать эту технику для любой величины, которая может быть выражена в виде вектора, включая скорость, импульс, силы, угловой момент и крутящий момент.

      Расширения этого метода

      В физике более высокого уровня вы увидите несколько других расширений этого метода: использование симметрии для уменьшения количества уравнений, которые вы должны учитывать, метод разделения переменных , и метод расширения собственной функции (вы можете рассматривать каждую собственную функцию, как если бы это было отдельное измерение). Поскольку я не хочу дублировать работу, а метод разделения переменных приводит непосредственно к разложению по собственным функциям, я приведу только пример метода симметрии.

      Пример: Баллистика

      Пренебрегая сопротивлением воздуха и предполагая, что кривизна Земли незначительна, какова максимальная дальность полета пушки с начальной скоростью v ?

      Мы можем рассмотреть размеры x и y по отдельности и использовать кинематические уравнения, чтобы найти ответ. Нам придется разделить скорость на компоненты x и y , используя синусоид 9.0007 и косинус . Обратите внимание, что поскольку у нас есть система из двух уравнений, у нас может быть два неизвестных, как я уже говорил ранее в разделе Explicit Equation . К сожалению, похоже, у нас есть три неизвестных, поскольку мы не знаем θ , угол, под которым стреляет пушка, но мы можем получить другое уравнение, поскольку пытаемся максимизировать дальность. Мы получим диапазон с точки зрения одного неизвестного, затем мы возьмем производную диапазона по этому неизвестному, установим ее на ноль, найдем неизвестное в новом уравнении, а затем выберем решение, которое дает нам максимальное значение.

      В этом случае мы знаем

      • начальное положение: (0, 0)
      • вектор начальной скорости (через другие переменные): ( v

        5

        6 , V SIN θ )

      • вектор ускорения (гравитация его вниз): (0, -G)

      We также знают также.0006 y , так как диапазон относится к тому, как далеко он пройдет, прежде чем упадет на землю, что происходит, когда y снова равно нулю.

      Два кинематических уравнения связывают начальную скорость и начальное ускорение с положением, и каждое имеет разные неизвестные. Если мы используем тот, который имеет конечную скорость как неизвестную (тот, который мы использовали ранее), мы получим

      в направлении x и

      в направлении y , где мы приняли отрицательное решение, поскольку пушечное ядро ​​движется вниз, когда оно падает на землю. Это кинематическое уравнение не помогло нам определить расстояние, потому что мы не могли использовать информацию из одного уравнения для решения другого и наоборот. Если вместо этого мы используем кинематическое уравнение, в котором время является неизвестным

      мы получаем

      , что означает, что мы можем получить ответ, решив t через θ или наоборот. Глядя на уравнение y, мы получаем

      , где мы выбрали ненулевое решение t , поскольку пушечное ядро ​​находится в точке (0, 0) при t = 0 . Поскольку у нас есть t в пересчете на θ , мы можем снова подставить это в уравнение для диапазона, что дает нам

      Теперь единственное неизвестное равно θ , поэтому мы хотим найти угол, который максимизирует дальность выстрела, что мы и делаем, используя процесс, описанный ранее в этом разделе (или используем тот факт, что максимум sin θ равен 1 и пропустите эту часть).

      Вы можете решить это уравнение несколькими способами, но я собираюсь сделать это чисто математическим способом (который станет более полезным в более поздних математике и физике, хотя для этой задачи это будет излишним). Я также собираюсь использовать радианы, потому что математика и физика, как правило, проще с радианами.

      Первое уравнение получается из того факта, что cos( θ ) имеет ноль в точке 1/2 π (90 градусов) и повторяет каждое целое число, кратное 7 π ), что и означает правая часть первого уравнения ( 1/2 π плюс некоторое целое число, кратное π ). Оттуда я делю обе стороны на 2 и получаю возможные значения θ . Поскольку θ должно находиться в диапазоне от 0 (стрельба прямо вперед) до 1/2 π (стрельба прямо вверх), единственное θ , которое максимально увеличивает расстояние полета ядра. 1/4 π (45 градусов).

      Подключив все, мы получим окончательный ответ:

      Пример: Balanced Forces

      Автомобиль движется вперед с постоянным ускорением. Внутри этой машины кольцо массой m в состоянии равновесия висит на натянутой нити под углом θ отличие от висения прямо вниз, как показано на схеме. Определить ускорение автомобиля.

      Сделано из тикза и латекса

      На кольцо действуют три силы:

      • ускорение автомобиля
      • сила тяжести
      • натяжение струны

      Поскольку мы имеем дело с силами, воздействующими на объект с известным суммарным ускорением (то есть 0, так как он находится в равновесии) и мы знаем направления всех сил, мы можем нарисовать диаграмму свободного тела.

      Обратите внимание, что сила от автомобиля полностью направлена ​​в направлении x , сила гравитации полностью направлена ​​в направлении y , а сила натяжения имеет компоненты x и y . Как и в предыдущей задаче, мы можем разбить силу натяжения на компоненты x и y , используя синус и косинус . Обратите внимание, что поскольку θ относится к положительной оси y , мы получаем y компонента натяжения путем умножения на cos(θ) , а не sin(θ) . Точно так же мы должны умножить на sin(θ) , чтобы получить составляющую напряжения x . Разбив эту систему на одно уравнение для каждого измерения, мы получим

      . Возможно, вы думаете, что мы застряли, потому что сейчас мы не знаем никаких сил, кроме гравитации (это -mg ). Мы можем попробовать кое-что, но мы можем сделать замену F = ma , для всех сил, и так как все силы действуют на кольцо, мы можем сделать следующее

      После этого мы найдем ускорение автомобиля и получим выражение через ускорение от напряжения и θ .

      Мы знаем θ , поэтому нам нужно другое уравнение, которое связывает ускорение от напряжения с чем-то другим, что мы знаем. В данном случае это θ и г . Подставив выражение для ускорения сзади в ускорение автомобиля, мы получим окончательный ответ g tan( θ ) с помощью описанного ниже процесса.

      Обратите внимание, что вам не нужно было использовать массу кольца, хотя она была указана в задаче. В реальном мире у вас часто будет больше информации, чем вам нужно, поэтому вам нужно выяснить, какую информацию использовать, а какую игнорировать.

      A Ярлык

      Поскольку сумма сил равна нулю, если сложить векторы встык, они образуют треугольник с силой натяжения в качестве гипотенузы, силой автомобиля в качестве противоположной стороны и силой тяжести в качестве противоположной стороны. смежная сторона, а это значит, что вы можете использовать определение касательной , чтобы получить ответ. Этот ярлык используется редко, но здесь он работает. Если вы не знаете, как использовать этот ярлык, не делайте этого и используйте процедуру, описанную выше.

      Пример: симметрия в электрических полях

      Каково электрическое поле на расстоянии r от бесконечно длинной линии заряда (т. е. провода) с однородной плотностью заряда λ при отсутствии тока?

      Эта проблема появится позже при изучении физики и потребует векторного исчисления для полного понимания, но общий принцип симметрии остается в силе.

      Если бы вы хотели, вы могли бы проинтегрировать по всему проводу и получить ответ, но закон Гаусса требует меньше работы (я могу посмотреть на эту задачу и сказать вам, что ответ λ/(2πεr) , потому что математика довольно проста с законом Гаусса, но я собираюсь пройти весь процесс, чтобы вы поняли, что происходит за кулисами). Однако, чтобы максимально эффективно использовать закон Гаусса, нам нужно найти полный набор поверхностей, которые мы можем использовать в любом месте провода, чтобы электрическое поле не менялось ни по величине, ни по направлению относительно поверхности. Вы можете подумать, что нам уже нужно знать электрическое поле, чтобы найти такую ​​поверхность, но мы можем использовать симметрию задачи, чтобы вычислить поверхность. Я собираюсь установить свою систему координат так, чтобы линия заряда проходила в z направление, которое вы можете себе представить вверх и вниз. Я также предполагаю, что на всех диаграммах заряд положительный (имеется в виду, что электрическое поле направлено в другую сторону), но если λ отрицательное, поменяйте направление всех стрелок на диаграмме.

      Так как линия заряда бесконечно длинна, не должно иметь значения, на какой участок провода мы смотрим. Каждая точка на проводе будет иметь одинаковое количество заряда на одинаковом расстоянии с обеих сторон.

      Этот факт дает нам два упрощения, которые мы можем сделать:

      • Провод имеет трансляционную симметрию вдоль его длины, что означает, что компонент провода z не имеет значения, и мы можем выбрать любой участок провода, чтобы нарисовать нашу поверхность.
      • Электрическое поле имеет нулевую составляющую z , поскольку распределение заряда симметрично в направлении z вокруг каждой точки.
      Это поле невозможно, так как оно имеет положительную z-компоненту. Все возможные поля не имеют z-компоненты.

      Мы также можем заметить, что вращение вокруг линии заряда не меняет распределения заряда, поэтому у нас также есть вращательная симметрия , что означает, что электрическое поле не может выглядеть как

      . Нарисовано с использованием numpy и pyplot.

      , где красная точка — провод, выходящий из экрана. Это также не может выглядеть как

      , потому что магнитное поле постоянно (в частности, 0), а уравнение Максвелла-Фарадея означает, что оно не имеет никакого завихрения. В этот момент электрическое поле может выглядеть только как

      с проводом, выходящим из экрана. Длина стрелок будет разной, но это правильное направление.

      Теперь мы знаем, что нам нужна поверхность, которую можно вращать вокруг проволоки или скользить по ней, что означает , что мы можем использовать цилиндр с центром вокруг проволоки в качестве нашей поверхности . Таким образом, закон Гаусса для нас в этом случае равен

      Заряд, содержащийся в цилиндре длиной L , равен всего лишь , так что правая часть уравнения решена. Общий поток через поверхность представляет собой сумму потоков через оба конца цилиндра и боковую часть цилиндра (там, где вы наклеили бы этикетку на банке). Включение их дает

      Поскольку электрическое поле перпендикулярно основаниям цилиндра (используйте нормали к поверхности для потока), они вносят нулевой поток.

      Поскольку электрическое поле направлено прямо из стороны цилиндра, скалярное произведение в интеграле просто становится двумерным интегралом, который представляет собой просто площадь поверхности стороны цилиндра. На данный момент мы можем получить ответ с помощью некоторой алгебры.

      С опытом вы научитесь распознавать цилиндрическую симметрию, после чего сможете без особых усилий перейти от закона Гаусса к математике.

      Хотя вы можете использовать силы и крутящие моменты, чтобы выяснить, как что-либо в классической механике будет двигаться, и, следовательно, вычислить другие связанные величины, это может быть проблемой. Если вы пытаетесь выяснить, насколько быстро мяч, катящийся по прямой рампе (без проскальзывания и сопротивления воздуха), движется в нижней части рампы, то вы можете использовать силы и крутящие моменты, даже если они требуют больше работы.

      Все силы везде одинаковы.

      Если вы пытаетесь определить, с какой скоростью мяч, катящийся по криволинейной рампе (без проскальзывания и сопротивления воздуха), движется в нижней части рампы с помощью сил и крутящих моментов, вам придется вычислить интеграл по траектории сила, которая меняется в зависимости от положения, так что удачи.

      AAAAAAHHHHHHH

      В лучшем случае, даже если вы получите решение, вы потратите на решение проблемы гораздо больше времени, чем нужно. Вместо этого поиск сохраняющихся количеств может дать вам ответ, который вы сможете быстро проверить. Три наиболее распространенные сохраняющиеся величины в задачах по физике I:

      • Линейный импульс: Отсутствие суммарных внешних сил на систему означает, что полный линейный импульс сохраняется.
      • Суммарная механическая энергия: Отсутствие сопротивления воздуха, проскальзывания, трения или повреждения объектов в системе, вероятно, означает сохранение полной механической энергии системы. Говоря более технически, если все силы в каждой точке образуют консервативное векторное поле, то энергия сохраняется. Если в задаче указано упругое столкновение, то сохраняется и полная механическая энергия.
      • Угловой момент: Что-то будет вращаться, и внешних крутящих моментов не будет.

      Если вы можете сохранить либо линейный, либо угловой момент, вы получите уравнение для каждого соответствующего измерения. Точно так же, если энергия сохраняется, вы получаете одно бесплатное уравнение, но это уравнение часто дает вам гораздо больше информации с меньшими усилиями.

      Пример: сохранение импульса и энергии

      Космонавт в космосе сбивает мяч в точке (-1, -3) с другим шаром той же массы в позиции (0, 0), который затем ударяется о стену в точке положение (2, 3). Перед столкновением первый шарик движется со скоростью ви . Каковы скорости обоих шаров сразу после столкновения? Предположим, что нет ни трения, ни сопротивления воздуха, ни гравитации, ни вращения.

      Механическая энергия сохраняется, так как задача говорит нам, что столкновение было упругим, так что мы получим уравнение. Мы также можем получить некоторые уравнения из закона сохранения импульса, поскольку после того, как космонавт ударит по первому мячу, внешние силы отсутствуют. У нас есть три неизвестных:

      • v1 , скорость первого мяча
      • v2 , скорость второго мяча
      • направление первого мяча после столкновения потому что он изменился с (-1, -3) на (0, 0) . Точно так же мы знаем направление, в котором летел второй мяч, потому что он прошел путь от (0, 0) до (2, 3) . Затем мы нормализуем эти векторы (разделив их на длину), чтобы в итоге мы получили только направление. над этими векторами, чтобы мы знали, что они являются единичными векторами.

        Мы прошли тот же процесс для v2 , что и для vi . Теперь, когда у нас есть единичные векторы, мы можем выразить векторы как произведение величины и направления.

        После этого я буду хранить все в переменных и буду работать с векторами, потому что это позволит мне работать с измерениями x и y одновременно, потому что математика довольно полезно. Сначала рассмотрим уравнение импульса.

        Теперь, когда у нас есть явное уравнение для v1 , мы можем посмотреть на уравнение полной механической энергии. Обратите внимание, что поскольку v1 — это вектор, мы можем поставить над ним точку, чтобы получить его величину в квадрате, что нам и понадобится для уравнения кинетической энергии.

        Поскольку скалярное произведение между векторами шляпы является скаляром (в частности, косинусом угла между vi и v2 ), я заменю его символом C , чтобы очистить математику.

        Теперь мы подставим это в уравнение полной механической энергии, и у нас должно получиться что-то, что мы сможем решить.

        Мы отвергаем решение v2 = 0 , поскольку знаем, что v2 не равно нулю, поэтому нам остается другое решение. Мы подставляем наш результат для v2 обратно в наше уравнение для v1 , и все готово.

        Пример: Энергосбережение

        Мяч массой m и радиусом R с одинаковой плотностью начинает катиться по пандусу (не обязательно плоскому) без сопротивления воздуха, без скольжения. Мяч имеет начальную скорость vi до того, как он достигнет рампы. Верхняя часть рампы находится на высоте h метров над нижней частью рампы. С какой скоростью движется мяч, когда он достигает нижней части пандуса?

        Иногда в этих задачах гораздо больше информации о рампе, но вам даже не понадобится все, что я дал вам в этой задаче. Поскольку нет ни сопротивления воздуха, ни скольжения, мы можем использовать сохранение энергии. У нас есть гравитационная потенциальная энергия, кинетическая энергия поступательного движения (масса и скорость) и кинетическая энергия вращения (инерция и угловая скорость) как в начале рампы, так и в нижней части рампы. По причинам слишком сложным, чтобы вдаваться в них, имеют значение только различия в потенциальной энергии , что означает, что мы можем произвольно сказать, что потенциальная энергия внизу равна 0, чтобы облегчить нашу математику.

        Поскольку он не скользит ни в одной точке, Rω = v в каждой точке, то есть у нас есть еще два уравнения, связывающие скорость и угловую скорость: одно в начале и одно в конце. Наконец, нам нужно знать момент инерции сферы, который мы можем найти либо путем прямого интегрирования, либо путем поиска.

        Подставив все в уравнение энергии, мы получим

        Мы можем исключить массу везде, так как она является частью каждого члена.

        Мы также можем использовать замену Rω = v , чтобы уменьшить количество переменных до одной неизвестной и трех заданных переменных. Затем мы можем найти конечную скорость. Весь процесс выглядит так:

        Вы можете быть немного сбиты с толку, потому что можете подумать, что конечная скорость должна зависеть от массы или радиуса объекта, но это не так. Помните, что в нашей ситуации нет сопротивления воздуха, и единственная энергия, добавляемая к сфере, исходит от гравитации, которая обеспечивает постоянное ускорение всем объектам независимо от массы. Точно так же больший радиус означает более высокий момент инерции, но, поскольку проскальзывания нет, угловая скорость, скорость и радиус ограничены таким образом, что кинетическая энергия вращения не зависит от радиуса.

        Нахождение обобщения

        Если мы допустим, что c будет отношением между моментом инерции и массой, умноженной на квадрат радиуса вращения, мы можем прийти к общему решению

        Если бы объект не имел момента инерции или не мог вращаться (скажем, коробка скользит по поверхности без трения), мы получили бы стандартное кинематическое уравнение

        , которое должно помочь вам чувствовать себя более уверенно в решении.

        У этой техники немного неправильное название, так как Знать фундаментальные отношения между различными величинами было бы более точным, но большинство этих отношений определяются в терминах производных (или эквивалентных интегралов), поэтому я решил выделить эту часть. Все пять уравнений кинематики могут быть получены из того факта, что скорость является производной по времени от положения, ускорение является производной по времени от скорости и предположением о постоянном ускорении. Другими важными соотношениями в физике I являются:

        • Сила — это производная по времени импульса и отрицательного градиента (общая пространственная производная, которая может быть выражена в различных системах координат и любом количестве измерений) потенциальной энергии.
        • Крутящий момент – это производная момента импульса по времени.
        • Работа в невращающейся системе представляет собой интеграл силы по траектории относительно положения.
        • Работа во вращающейся системе представляет собой интеграл по траектории крутящего момента относительно углового положения.

        Поскольку я использовал эту технику во всех своих примерах, я не буду приводить пример этого руководства.

        Хранение всего в переменных имеет несколько преимуществ:

        • Вы можете сказать, имеет ли ответ смысл, изменив заданные значения до их логических крайностей. Например, если сила тяжести увеличивается по мере удаления от массы, вы, вероятно, где-то допустили ошибку.
        • Гораздо сложнее найти, какой конкретный шаг привел к ошибке в серии арифметических операций, чем найти ошибку в серии алгебраических операций.
        • Переменные часто сокращаются, оставляя вам более простое уравнение и меньше работы.
        • Как и в разделе Продвинутый прием , в обоих примерах в разделе Взгляд на каждое измерение индивидуально и в примере Энергосбережение , вы часто можете найти части выражения, которые можно преобразовать во что-то, что вы уже знаете и уменьшите количество арифметических действий, которые вам нужно сделать.

        Вы можете обобщить этот принцип на такие вещи, как не расширять все до тех пор, пока вам это не нужно, например, не разбивать вещи на отдельные измерения, пока у вас не будет ваших уравнений, не разлагать факторизованные полиномы и т. д. Вы также должны искать упрощения, которые вы можете сделать в математике, будь то факторизация переменных, обезразмеривание переменных, введение констант и т. д.

        Эти методы или концепции либо слишком специфичны, чтобы уделить им целый раздел, либо не нуждаются в полном объяснении, поэтому я собираюсь перечислить их здесь.

        • Натяжение — это передача силы с одной стороны веревки/цепи на другую, поэтому вы можете сложить все силы с одной стороны веревки, получить ее величину, сложить все силы с другой стороны веревки. веревку, получить ее величину, а затем найти разницу, чтобы найти результирующую силу.
        • Волны, поток/давление жидкости и колебательные движения, такие как маятник (множественное число среднего рода оканчивается на «а» на латыни), пружины и орбиты на уровне физики I описываются алгебраическими уравнениями, но на более поздних занятиях они становятся дифференциальными уравнениями. А пока придерживайтесь алгебраических версий.
        • Физика — это в основном математика, основанная на нескольких конкретных фактах и ​​уравнениях. Изучение математики даст вам новые методы, ярлыки и более глубокое понимание физики. Для базовой физики (импульс, энергия, силы, крутящие моменты и т. д.) я предлагаю изучить некоторые основы линейной алгебры (векторы, скалярные произведения и перекрестные произведения), тригонометрию и исчисление с одной переменной. Для основ электромагнетизма я предлагаю изучить исчисление с несколькими переменными (обычно исчисление III в США). Для всего, что после этого, я предлагаю изучить обыкновенные дифференциальные уравнения и уравнения в частных производных.
        • Будьте последовательны . Если вы решите, что положительная ось y будет направлена ​​вниз, то гравитация обеспечит положительное ускорение. Если вы решите, что W — это работа, выполненная в системе, то U = Q+W . В общем, как только вы определились с настройкой, вы можете, не задумываясь, посчитать результаты и интерпретировать результаты на основе вашей настройки.

        Угловая кинематика

        Угловые величины были моими наименее любимыми частями физики I, потому что их задачи были линейными задачами с несколькими дополнительными шагами или уравнениями, поэтому я избегал использования задач с угловыми величинами в качестве примеров, чтобы не допустить этой статьи. быть слишком длинным.

        • Все угловые кинематические величины (угловое расстояние, угловая скорость и угловое ускорение) представляют собой нормальные линейные величины, деленные на r , или расстояние от оси вращения, и имеют одни и те же кинематические уравнения.
        • Угловой момент, угловая скорость и крутящий момент — все это векторы (псевдовекторы по-прежнему являются элементами векторного пространства), направление которых является осью вращения, поэтому вы можете делать такие вещи, как складывать их вместе и разделять на отдельные измерения.
        • При решении проблем с крутящим моментом, если вы не знаете силы в точке, попробуйте найти крутящий момент вокруг этой точки, поскольку неизвестные силы будут вносить нулевой крутящий момент. Например, лестница, прислоненная к стене, будет воздействовать на стену силой, но вы, вероятно, не сможете вычислить эту силу, не решив всей задачи, поэтому найдите крутящий момент в точке, где лестница касается стены.

        Выясните, что работает для вас, что не работает для вас, что работает для других, и причины всех трех. Если что-то работает для кого-то другого, но не для вас, подумайте, стоит ли заставить это работать для вас, и подумайте, что вам нужно сделать, чтобы это сработало для вас. Например, я предпочитаю использовать более мощную математику, потому что это позволяет мне поставить задачу и позволить математике сделать всю работу за меня, и вы можете извлечь выгоду из такого подхода.

        Хотя я могу поделиться с вами тем, что знаю, лучшие техники и рекомендации — это те, которые вы сами вырабатываете своими ошибками, усилиями и достижениями. Если вы хотите преуспеть в физике, возьмите то, что я вам дал, и сделайте это своим.

        Если вы изучали или изучали физику, оставьте в комментариях ниже любые методы и рекомендации, которые вам помогли.

        Я подумываю о создании серии статей в похожем формате и стиле по другим темам, поэтому, если вам понравилась эта статья, следите за обновлениями.

        Отказ от ответственности

        Я написал все примеры задач на основе описанных методов и рекомендаций. Некоторые примеры основаны на хорошо известных задачах, таких как дальность полета баллистического снаряда и объект, висящий на веревке в ускоряющемся транспортном средстве, или они настолько просты, что вполне вероятно, что кто-то еще написал подобную задачу, например пример дано в разделе Продвижение вперед на любом фронте, каким бы малым он ни был .

        Я сделал все изображения с помощью LaTeX, tikz и GIMP.

        Кинематические уравнения: Примеры задач и решений

        Ранее в Уроке 6 были введены и обсуждены четыре кинематических уравнения. Для использования с этими уравнениями была представлена ​​полезная стратегия решения проблем, и были приведены два примера, иллюстрирующие использование этой стратегии. Затем было обсуждено и проиллюстрировано применение кинематических уравнений и стратегии решения задач к свободному падению. В этой части Урока 6 будет представлено несколько примеров задач. Эти задачи позволяют любому изучающему физику проверить свое понимание использования четырех кинематических уравнений для решения задач, связанных с одномерным движением объектов. Вам предлагается прочитать каждую задачу и попрактиковаться в использовании стратегии при решении задачи. Затем нажмите кнопку, чтобы проверить ответ, или воспользуйтесь ссылкой для просмотра решения.

        Проверьте свое понимание

        1. Самолет разгоняется по взлетно-посадочной полосе со скоростью 3,20 м/с 2 в течение 32,8 с, пока, наконец, не оторвется от земли. Определить расстояние, пройденное до взлета.
        2. Автомобиль трогается с места из состояния покоя и равномерно разгоняется за время 5,21 секунды на расстояние 110 м. Определить ускорение автомобиля.
        3. Аптон Чак едет на Гигантской капле в Большой Америке. Если Аптон будет свободно падать в течение 2,60 с, какова будет его конечная скорость и как далеко он упадет?
        4. Гоночный автомобиль равномерно ускоряется с 18,5 м/с до 46,1 м/с за 2,47 секунды. Определить ускорение автомобиля и пройденный путь.
        5. Перышко падает на Луну с высоты 1,40 метра. Ускорение свободного падения на Луне равно 1,67 м/с 2 . Определите время падения пера на поверхность Луны.

          См. решение ниже.


        6. Сани с реактивным двигателем используются для проверки реакции человека на ускорение. Если сани с ракетным двигателем разгоняются до скорости 444 м/с за 1,83 секунды, то каково ускорение и какое расстояние проходят сани?
        7. Велосипед равномерно ускоряется из состояния покоя до скорости 7,10 м/с на пути 35,4 м. Определить ускорение велосипеда.
        8. Инженер проектирует взлетно-посадочную полосу для аэропорта. Из самолетов, которые будут использовать аэропорт, наименьшая скорость ускорения, вероятно, составит 3 м/с 2 . Взлетная скорость этого самолета составит 65 м/с. Предполагая это минимальное ускорение, какова минимальная допустимая длина взлетно-посадочной полосы?
        9. Автомобиль, движущийся со скоростью 22,4 м/с, останавливается за 2,55 с. Определить расстояние заноса автомобиля (считая ускорение равномерным).
        10. Кенгуру способен прыгать на высоту 2,62 м. Определить скорость взлета кенгуру.
        11. Если у Майкла Джордана вертикальный прыжок 1,29 м, то какова его скорость взлета и время зависания (общее время, чтобы подняться вверх до пика и затем вернуться на землю)?
        12. Пуля вылетает из винтовки с начальной скоростью 521 м/с. Разгоняясь через ствол винтовки, пуля проходит расстояние 0,840 м. Определить ускорение пули (ускорение считать равномерным).
        13. Бейсбольный мяч подбрасывается прямо в воздух, и его время зависания составляет 6,25 с. Определите высоту, на которую поднимается мяч, прежде чем он достигнет своей вершины. (Подсказка: время подъема на пик составляет половину общего времени зависания. )
        14. Смотровая площадка высотного небоскреба высотой 370 м над ул. Определите время, за которое монета свободно упадет с палубы на улицу.

          См. решение ниже.


        15. Пуля, летящая со скоростью 367 м/с, застревает в комке влажной глины. Пуля проникает на расстояние 0,0621 м. Определить ускорение пули при движении в глину. (Предположим, что ускорение равномерное.)
        16. Камень брошен в глубокий колодец, и слышно, как он ударяется о воду через 3,41 с после падения. Определить глубину скважины.
        17. Однажды было зафиксировано, что Jaguar оставил следы заноса длиной 290 метров. Предполагая, что Ягуар занесло до остановки с постоянным ускорением -3,90 м/с 2 , определите скорость Ягуара до начала заноса.
        18. Самолет имеет скорость взлета 88,3 м/с, и для достижения этой скорости требуется 1365 м. Определить ускорение самолета и время, необходимое для достижения этой скорости.
        19. Драгстер разгоняется до скорости 112 м/с на расстоянии 398 м. Определить ускорение (предположим равномерным) драгстера.
        20. С какой скоростью в милях/ч (1 м/с = 2,23 мили/ч) должен быть брошен предмет, чтобы он достиг высоты 91,5 м (эквивалентно одному футбольному полю)? Примите пренебрежимо малое сопротивление воздуха.

        Решения вышеуказанных проблем

        1. Дано:

          а = +3,2 м/с 2

          т = 32,8 с

          v i = 0 м/с

          Найти:

          д = ??
          d = v i *t + 0,5*a*t 2

          d = (0 м/с)*(32,8 с)+ 0,5*(3,20 м/с 2 )*(32,8 с) 2

          д = 1720 м

          Вернуться к проблеме 1

           

        2. Дано:

          д = 110 м

          т = 5,21 с

          v i = 0 м/с

          Найти:

          а = ??
          d = v i *t + 0,5*a*t 2

          110 м = (0 м/с)*(5,21 с)+ 0,5*(а)*(5,21 с) 2

          110 м = (13,57 с 2 )*a

          а = (110 м)/(13,57 с 2 )

          а = 8,10 м/с 2

          Вернуться к задаче 2

           

        3. Дано:

          а = -9,8 м

          т = 2,6 с

          v i = 0 м/с

          Найти:

          д = ??

          v f = ??

          d = v i *t + 0,5*a*t 2

          d = (0 м/с)*(2,60 с)+ 0,5*(-9,8 м/с 2 )*(2,60 с) 2

          d = -33,1 м (- указывает направление)

          v f = v i + a*t

          v f = 0 + (-9,8 м/с 2 )*(2,60 с)

          v f = -25,5 м/с (- указывает направление)

          Вернуться к проблеме 3

           

        4. Дано:

          v i = 18,5 м/с

          v f = 46,1 м/с

          т = 2,47 с

          Найти:

          д = ??

          а = ??

          а = (Дельта v)/t

          а = (46,1 м/с — 18,5 м/с)/(2,47 с)

          а = 11,2 м/с 2

          д = v i *t + 0,5*a*t 2

          d = (18,5 м/с)*(2,47 с)+ 0,5*(11,2 м/с 2 )*(2,47 с) 2

          d = 45,7 м + 34,1 м

          д = 79,8 м

          (Примечание: d также можно рассчитать с помощью уравнения v f 2 = v i 2 + 2*a*d)

          Вернуться к задаче 4

           

        5. Дано:

          v i = 0 м/с

          d = -1,40 м

          а = -1,67 м/с 2

          Найти:

          т = ??
          d = v i *t + 0,5*a*t 2

          -1,40 м = (0 м/с)*(t)+ 0,5*(-1,67 м/с 2 )*(t) 2

          -1,40 м = 0+ (-0,835 м/с 2 )*(t) 2

          (-1,40 м)/(-0,835 м/с 2 ) = t 2

          1,68 с 2 = т 2

          т = 1,29 с

          Вернуться к проблеме 5

           

        6. Дано:

          v i = 0 м/с

          v f = 444 м/с

          т = 1,83 с

          Найти:

          а = ??

          д = ??

          а = (Дельта v)/t

          а = (444 м/с — 0 м/с)/(1,83 с)

          а = 243 м/с 2

          d = v i *t + 0,5*a*t 2

          d = (0 м/с)*(1,83 с)+ 0,5*(243 м/с 2 )*(1,83 с) 2

          d = 0 м + 406 м

          д = 406 м

          (Примечание: d также можно рассчитать с помощью уравнения v f 2 = v i 2 + 2*a*d)

          Вернуться к задаче 6

           


           

        7. Дано:

          v i = 0 м/с

          v f = 7,10 м/с

          д = 35,4 м

          Найти:

          а = ??
          v f 2 = v i 2 + 2*a*d

          (7,10 м/с) 2 = (0 м/с) 2 + 2*(а)*(35,4 м)

          50,4 м 2 2 = (0 м/с) 2 + (70,8 м)*a

          (50,4 м 2 2 )/(70,8 м) = а

          а = 0,712 м/с 2

          Вернуться к задаче 7

           

        8. Дано:

          v i = 0 м/с

          v f = 65 м/с

          а = 3 м/с 2

          Найти:

          д = ??
          v f 2 = v i 2 + 2*a*d

          (65 м/с) 2 = (0 м/с) 2 + 2*(3 м/с 2 )*д

          4225 м 2 2 = (0 м/с) 2 + (6 м/с 2 )*d

          (4225 м 2 2 )/(6 м/с 2 ) = d

          д = 704 м

          Вернуться к задаче 8

           

        9. Дано:

          v i = 22,4 м/с

          v f = 0 м/с

          т = 2,55 с

          Найти:

          д = ??
          d = (v i + v f )/2 *t

          d = (22,4 м/с + 0 м/с)/2 * 2,55 с

          d = (11,2 м/с)*2,55 с

          д = 28,6 м

          Вернуться к задаче 9

           

        10. Дано:

          а = -9,8 м/с 2

          v f = 0 м/с

          д = 2,62 м

          Найти:

          v i = ??
          v f 2 = v i 2 + 2*a*d

          (0 м/с) 2 = v i 2 + 2*(-9,8 м/с 2 )*(2,62 м)

          0 м 2 2 = v i 2 — 51,35 м 2 2

          51,35 м 2 2 = v i 2

          v i = 7,17 м/с

          Вернуться к проблеме 10

           

        11. Дано:

          а = -9,8 м/с 2

          v f = 0 м/с

          д = 1,29 м

          Найти:

          v i = ??

          т = ??

          v f 2 = v i 2 + 2*a*d

          (0 м/с) 2 = v i 2 + 2*(-9,8 м/с 2 )*(1,29 м)

          0 м 2 2 = v i 2 — 25,28 м 2 2

          25,28 м 2 2 = v i 2

          v i = 5,03 м/с

          Чтобы найти время зависания, найдите время до пика и удвойте его.

          против f = v i + a*t

          0 м/с = 5,03 м/с + (-9,8 м/с 2 )*t до

          -5,03 м/с = (-9,8 м/с 2 )*t до

          (-5,03 м/с)/(-9,8 м/с 2 ) = t up

          т до = 0,513 с

          время зависания = 1,03 с

          Вернуться к задаче 11

           

        12. Дано:

          v i = 0 м/с

          v f = 521 м/с

          d = 0,840 м

          Найти:

          а = ??
          v f 2 = v i 2 + 2*a*d

          (521 м/с) 2 = (0 м/с) 2 + 2*(а)*(0,840 м)

          271441 м 2 2 = (0 м/с) 2 + (1,68 м)*a

          (271441 м 2 2 )/(1,68 м) = а

          а = 1,62*10 5 м/с 2

          Вернуться к задаче 12

           

        13. Дано:

          а = -9,8 м/с 2

          v f = 0 м/с

          т = 3,13 с

          Найти:

          д = ??
          1. (ПРИМЕЧАНИЕ: время выхода на пик траектории составляет половину общего времени зависания — 3,125 с. )

          Первое использование: v f  = v i  + a*t

          0 м/с = v i  + (-9,8 м/с 2 )*(3,13 с)

          0 м/с = v i  — 30,7 м/с

          v i = 30,7 м/с (30,674 м/с)

          Теперь используйте: v f 2  = v i 2  + 2*a*d

          (0 м/с) 2  = (30,7 м/с) 2  + 2*(-9,8 м/с 2 )*(d)

          0 м 2 2 = (940 м 2 2 ) + (-19,6 м/с 2 )*d

          -940 м 2 2  = (-19,6 м/с 2 )*d

          (-940 м 2 2 )/(-19,6 м/с 2 ) = d

          д = 48,0 м

          Вернуться к задаче 13

           

        14. Дано:

          v i = 0 м/с

          д = -370 м

          а = -90,8 м/с 2

          Найти:

          т = ??
          d = v i *t + 0,5*a*t 2

          -370 м = (0 м/с)*(t)+ 0,5*(-9,8 м/с 2 )*(t) 2

          -370 м = 0+ (-4,9 м/с 2 )*(t) 2

          (-370 м)/(-4,9 м/с 2 ) = t 2

          75,5 с 2 = т 2

          т = 8,69 с

          Вернуться к задаче 14

           

           

        15. Дано:

          v i = 367 м/с

          против f = 0 м/с

          d = 0,0621 м

          Найти:

          а = ??
          v f 2 = v i 2 + 2*a*d

          (0 м/с) 2 = (367 м/с) 2 + 2*(а)*(0,0621 м)

          0 м 2 2 = (134689 м 2 2 ) + (0,1242 м)*a

          -134689 м 2 2 = (0,1242 м)*a

          (-134689 м 2 2 )/(0,1242 м) = а

          а = -1,08*10 6 м/с 2

          (Знак — означает, что пуля замедлилась. )

          Вернуться к задаче 15

           

        16. Дано:

          а = -9,8 м/с 2

          т = 3,41 с

          v i = 0 м/с

          Найти:

          д = ??
          d = v i *t + 0,5*a*t 2

          d = (0 м/с)*(3,41 с)+ 0,5*(-9,8 м/с 2 )*(3,41 с) 2

          d = 0 м+ 0,5*(-9,8 м/с 2 )*(11,63 с 2 )

          d = -57,0 м

          (ПРИМЕЧАНИЕ: знак — указывает направление)

          Вернуться к задаче 16

           

        17. Дано:

          а = -3,90 м/с 2

          v f = 0 м/с

          д = 290 м

          Найти:

          в и = ??
          v f 2 = v i 2 + 2*a*d

          (0 м/с) 2 = v i 2 + 2*(-3,90 м/с 2 )*(290 м)

          0 м 2 2 = v i 2 — 2262 м 2 2

          2262 м 2 2 = v i 2

          v i = 47,6 м/с

          Вернуться к задаче 17

           

        18. Дано:

          v i = 0 м/с

          v f = 88,3 м/с

          д = 1365 м

          Найти:

          а = ??

          т = ??

          v f 2 = v i 2 + 2*a*d

          (88,3 м/с) 2 = (0 м/с) 2 + 2*(а)*(1365 м)

          7797 м 2 2 = (0 м 2 2 ) + (2730 м)*a

          7797 м 2 2 = (2730 м)*a

          (7797 м 2 2 )/(2730 м) = а

          а = 2,86 м/с 2

          v f = v i + a*t

          88,3 м/с = 0 м/с + (2,86 м/с 2 )*t

          (88,3 м/с)/(2,86 м/с 2 ) = t

          t = 30,8 с

          Вернуться к задаче 18

           

        19. Дано:

          v i = 0 м/с

          v f = 112 м/с

          д = 398 м

          Найти:

          а = ??
          v f 2 = v i 2 + 2*a*d

          (112 м/с) 2 = (0 м/с) 2 + 2*(а)*(398 м)

          12544 м 2 2 = 0 м 2 2 + (796 м)*a

          12544 м 2 2 = (796 м)*а

          (12544 м 2 2 )/(796 м) = а

          а = 15,8 м/с 2

          Вернуться к проблеме 19

           

        20. Дано:

          а = -9,8 м/с 2

          v f = 0 м/с

          д = 91,5 м

          Найти:

          v i = ??

          т = ??

          Сначала найдите скорость в м/с:

          v f 2 = v и 2 + 2*а*д

          (0 м/с) 2 = v i 2 + 2*(-9,8 м/с 2 )*(91,5 м)

          0 м 2 2 = v i 2 — 1793 м 2 2

          1793 м 2 2 = v i 2

          v i = 42,3 м/с

          Теперь конвертируем из м/с в мили/ч:

          v i = 42,3 м/с * (2,23 мили/ч)/(1 м/с)

          v i = 94,4 мили/ч

          Вернуться к проблеме 20

         

        Следующий раздел:

        1.

        8: Решение задач по физике
        1. Последнее обновление
        2. Сохранить как PDF
      • Идентификатор страницы
        4304
        • OpenStax
        • OpenStax
        Цели обучения
        • Описать процесс разработки стратегии решения проблем.
        • Объясните, как найти численное решение задачи.
        • Кратко опишите процесс оценки значимости численного решения задачи.

        Навыки решения задач явно необходимы для успеха в количественном курсе физики. Что еще более важно, способность применять общие физические принципы, обычно представляемые уравнениями, к конкретным ситуациям является очень мощной формой знания. Это намного мощнее, чем запоминание списка фактов. Аналитические навыки и способность решать проблемы могут быть применены к новым ситуациям, в то время как список фактов не может быть достаточно длинным, чтобы охватить все возможные обстоятельства. Такие аналитические навыки полезны как для решения задач в этом тексте, так и для применения физики в повседневной жизни.

        .Рис. \(\PageIndex{1}\): Навыки решения задач необходимы для успеха в физике. (кредит: «scui3asteveo»/Flickr)

        Как вы, вероятно, хорошо знаете, для решения проблем требуется определенный творческий подход и проницательность. Никакая жесткая процедура не работает каждый раз. Креативность и проницательность растут с опытом. С практикой основы решения проблем становятся почти автоматическими. Один из способов попрактиковаться — прорабатывать примеры текста для себя во время чтения. Другой вариант — проработать как можно больше задач в конце раздела, начиная с самых простых, чтобы укрепить уверенность, а затем переходя к более сложным. После того, как вы начнете заниматься физикой, вы увидите ее вокруг себя и сможете начать применять ее к ситуациям, с которыми сталкиваетесь за пределами классной комнаты, точно так же, как это делается во многих приложениях в этом учебнике.

        Хотя простого пошагового метода, подходящего для решения каждой проблемы, не существует, описанный ниже трехэтапный процесс облегчает решение проблемы и делает его более осмысленным. Три этапа — это стратегия, решение и значимость. Этот процесс используется в примерах по всей книге. Здесь мы рассмотрим каждый этап процесса по очереди.

        Стратегия

        Стратегия — это начальный этап решения проблемы. Идея состоит в том, чтобы точно выяснить, в чем заключается проблема, а затем разработать стратегию ее решения. Вот некоторые общие рекомендации для этого этапа:

        • Изучите ситуацию, чтобы определить, какие физические принципы задействованы . Часто бывает полезно сначала нарисовать простой эскиз . Вам часто нужно решить, какое направление является положительным, и отметить это на своем эскизе. Когда вы определили физические принципы, гораздо легче найти и применить уравнения, представляющие эти принципы. Хотя найти правильное уравнение очень важно, имейте в виду, что уравнения представляют собой физические принципы, законы природы и отношения между физическими величинами. Без концептуального понимания проблемы численное решение не имеет смысла.
        • Составьте список того, что дано или может быть выведено из заявленной проблемы (укажите «известные») . Многие проблемы сформулированы очень кратко и требуют некоторого изучения, чтобы определить, что известно. На этом этапе очень полезно также нарисовать эскиз. Формальная идентификация известных имеет особое значение при применении физики к реальным ситуациям. Например, слово «остановился» означает, что в данный момент скорость равна нулю. Кроме того, мы часто можем принять начальное время и положение за ноль, выбрав соответствующую систему координат.
        • Точно определите, что нужно определить в задаче (идентифицируйте неизвестные). Особенно в сложных задачах не всегда очевидно, что нужно найти и в какой последовательности. Составление списка может помочь выявить неизвестные.
        • Определите, какие физические принципы могут помочь вам решить проблему . Поскольку физические принципы имеют тенденцию выражаться в виде математических уравнений, здесь может помочь список известных и неизвестных. Легче всего, если вы сможете найти уравнения, содержащие только одно неизвестное, то есть все остальные переменные известны, так что вы сможете легко найти неизвестное. Если уравнение содержит более одного неизвестного, то для решения задачи необходимы дополнительные уравнения. В некоторых задачах необходимо определить несколько неизвестных, чтобы получить наиболее необходимое. В таких задачах особенно важно помнить о физических принципах, чтобы не заблудиться в море уравнений. Возможно, вам придется использовать два (или более) разных уравнения, чтобы получить окончательный ответ.

        Решение

        Стадия решения — это когда вы делаете математику. Подставьте известные величины (вместе с их единицами измерения) в соответствующее уравнение и получите численное решение с единицами измерения . То есть проделайте алгебру, исчисление, геометрию или арифметику, необходимые для нахождения неизвестного из известного, убедившись, что в вычислениях используются единицы измерения. Этот шаг явно важен, потому что он дает числовой ответ вместе с его единицами измерения. Обратите внимание, однако, что этот этап составляет лишь одну треть всего процесса решения проблемы.

        Значение

        После выполнения математических расчетов на этапе решения проблемы возникает искушение подумать, что вы закончили. Но всегда помните, что физика — это не математика. Скорее, занимаясь физикой, мы используем математику как инструмент, помогающий нам понять природу. Итак, после получения числового ответа всегда следует оценивать его значимость:

        • Проверьте свои единицы . Если единицы ответа неверны, значит, была допущена ошибка, и вам следует вернуться к предыдущим шагам, чтобы найти ее. Один из способов найти ошибку — проверить все уравнения, которые вы вывели, на согласованность размеров. Однако имейте в виду, что правильные единицы измерения не гарантируют правильность числовой части ответа.
        • Проверьте ответ, чтобы убедиться, что он разумен. Имеет ли это смысл? Этот шаг чрезвычайно важен: – цель физики – точно описать природу. Чтобы определить, является ли ответ разумным, проверьте как его величину, так и его знак, в дополнение к его единицам измерения. Величина должна соответствовать приблизительной оценке того, какой она должна быть. Его также следует разумно сравнивать с величинами других величин того же типа. Знак обычно говорит вам о направлении и должен соответствовать вашим прежним ожиданиям. Ваше суждение улучшится по мере того, как вы будете решать больше физических задач, и у вас появится возможность делать более точные суждения о том, адекватно ли природа описана ответом на задачу. Этот шаг возвращает проблеме ее концептуальное значение. Если вы можете судить о том, является ли ответ разумным, у вас есть более глубокое понимание физики, чем просто возможность решить проблему механически.
        • Проверьте, не говорит ли ответ вам что-нибудь интересное. Что это значит? Это обратная сторона вопроса: есть ли смысл? В конечном счете, физика — это понимание природы, и мы решаем физические задачи, чтобы немного узнать о том, как работает природа. Поэтому, предполагая, что ответ действительно имеет смысл, вы всегда должны уделить время тому, чтобы посмотреть, говорит ли он вам что-то о мире, что вы находите интересным. Даже если ответ на эту конкретную проблему вам не очень интересен, как насчет метода, который вы использовали для ее решения? Можно ли адаптировать метод для ответа на интересующий вас вопрос? Во многих отношениях именно в ответах на подобные вопросы наука прогрессирует.

        Эта страница под названием 1.8: Решение задач по физике распространяется под лицензией CC BY 4.0 и была создана, изменена и/или курирована OpenStax с использованием исходного контента, который был отредактирован в соответствии со стилем и стандартами платформы LibreTexts; подробная история редактирования доступна по запросу.

        1. Наверх
          • Была ли эта статья полезной?
          1. Тип изделия
            Раздел или страница
            Автор
            ОпенСтакс
            Лицензия
            СС BY
            Версия лицензии
            4,0
            Программа ООР или издатель
            ОпенСтакс
            Показать оглавление
            нет
          2. Теги
            1. source@https://openstax. org/details/books/university-physics-volume-1

          Решение задач по физике

          Решение задач по физике

          РЕШЕНИЕ ЗАДАЧ ПО ФИЗИКЕ СТРАТЕГИЯ
          Доктор Марк Холлабо
          Общественный колледж Нормандейла

          http://www.nr.cc.mn.us/physics/Faculty/HOLLABGH/probsolv.htm

              Два фактора помогут вам стать лучше решение задач по физике. Прежде всего, вы должны знать и понимать принципы физики. Во-вторых, вы должны иметь стратегию применения этих принципов в новых ситуациях в чем может помочь физика. Мы называем эти ситуации проблемы. Многие студенты говорят: «Я понимаю материала, я просто не могу решать задачи. Если это верно для вас как студента-физика, то, возможно, вам нужно развивать свои навыки решения проблем. Имея стратегию, чтобы организовать эти навыки могут помочь вам.

              Решению задач по физике можно научиться, просто Например, вы научились водить машину, играть на музыкальном инструменте или кататься на велосипеде. Что может помочь вам больше всего, так это иметь общий подход к каждой проблеме, которую вы сталкиваться. Вы можете использовать различные инструменты или тактики с разных областях физики, но общая стратегия остается такой же. Скорее всего, вы уже приобрели некоторые навыки решения проблем и привычки из предыдущих курсов в физики, химии или математики. Как и в других областях обучения и жизни, некоторые из этих привычек могут быть полезными и некоторые могут на самом деле помешать вашему прогрессу в изучении того, как решать задачи по физике.

              Итак, изучая этот новый подход, будьте готовность пробовать новые идеи и отказываться от старых привычек, которые могут факт будет препятствовать вашему пониманию. По мере взросления как решения физических задач, вы обнаружите, что подход будет стать для вас второй натурой. Вы начнете автоматически делать те вещи, которые приведут вас к созданию эффективной Решение проблемы.

              Как и во многих других учебных мероприятиях, полезно разбить стратегию решения проблем на основные и мелкие шаги. Стратегия, которую мы хотели бы, чтобы вы узнали, имеет пять основных шагов:  Сосредоточьтесь на проблеме , Физика Описание , Планирование решения , Выполнение плана , и Оценка решения . Давайте возьмем подробно рассмотрите каждый из этих шагов, а затем сделайте образец проблема после стратегии. На данном этапе нашего обсуждение, не волнуйтесь, если есть термины или понятия физики что вы не понимаете. Вы узнаете эти понятия как они нужны. Затем вернитесь к этому обсуждению.

          АКЦЕНТ НА ​​ПРОБЛЕМУ
          Обычно, когда вы читаете заявление задача физики, вы должны визуализировать задействованные объекты и их контекст. Вам нужно нарисовать рисунок и указать любой предоставленная информация.


          (1) Сначала создайте мысленный образ проблемной ситуации.
          (2) Затем нарисуйте грубый, хотя и буквальный, рисунок, показывающий важные объекты, их движение и их взаимодействия. Взаимодействие, например, может состоять один объект связан с другим веревкой.
          (3) Отметьте всю известную информацию. В этот момент не беспокоиться о присвоении алгебраических символов конкретным количества.

              Иногда вопрос, заданный в проблема не очевидна. Верёвка безопасна? не то, что вы можете прямо ответить. Спросите себя, что конкретно просят? Как это переводится в какая-то исчисляемая величина?

              Есть много способов решить физический проблема. Одна часть обучения тому, как решить проблему, состоит в том, чтобы знать, какой подход использовать. Вам нужно будет наметить концепции и принципы, которые, по вашему мнению, будут полезны при решении проблема.


          Если задействованы простые движения, используйте кинематику определение скорости и ускорения.
          Если задействованы силы и объекты взаимодействуют из-за этих силы, используйте законы движения Ньютона.
          Силы, которые действуют в течение определенного промежутка времени и заставляют объекты изменить их скорость предлагает использовать сохранение импульса.
          Часто в ситуациях, связанных с теплофизикой или электромагнетизм, принцип сохранения энергии Полезно.
          Возможно, вам потребуется указать временные интервалы, в течение которых применение каждого принципа будет наиболее полезным.
          Важно выявить любые ограничения, присутствующие в такая ситуация, как «машина не уезжает» дорога .
          Укажите любые приближения или упрощения, которые вы думаете облегчит решение проблемы, но не существенно повлиять на результат. Часто мы не учитывать силы трения из-за сопротивления воздуха.

              Ваш подход, вероятно, будет очень последовательно на протяжении всего раздела учебника. Задача для вас будет заключаться в применении подхода в различных ситуаций.

          ОПИСАТЬ ФИЗИКУ
          «Физическое описание» проблема переводит данную информацию и очень буквальный изображение в идеализированную диаграмму и определяет переменные, которые могут манипулировать для расчета желаемых величин. В некотором смысле, вы переводите буквальную ситуацию в идеализированную ситуации, когда вы можете применить законы физики. Самым большим недостатком начинающих решателей физических задач является пытаясь применить законы физики, то есть записать уравнений, прежде чем приступить к качественному анализу проблема. Если вы сможете устоять перед искушением поиска уравнения слишком рано в вашем решении проблемы, вы станете гораздо более эффективное решение проблем.

          Чтобы составить описание физики, вы должны выполнить далее:

          • Превратите свое изображение в диаграмму (диаграммы), которая дает только необходимая информация для математического решение. На идеализированной диаграмме люди, машины, а другие объекты могут стать квадратными блоками или точками.
          • Определите символ для каждой важной физической переменной на ваша схема.
          • Обычно вам нужно нарисовать систему координат, показывающую + и — направления.
          • Если вы используете концепции кинематики, нарисуйте движение диаграмма, определяющая скорость и ускорение в определенных положениях и временах.
          • Если важны взаимодействия, нарисуйте идеализированное свободное тело, и силовые диаграммы.
          • При использовании принципов сохранения рисуйте «до», «перенос» (т.е. во время), и диаграммы «после», чтобы показать, как система изменения. Рядом с вашей диаграммой (диаграммами) укажите значение для каждой физической переменной, которую вы пометили на диаграммы или указать, что она неизвестна.

              Затем, используя вопрос, ваша физика описание и подход, который вы изложили, вам нужно будет определить целевую переменную. То есть вы должны решить, что неизвестное количество — это то, что вы должны вычислить из своего списка переменные. Спросите себя, отвечает ли рассчитанное количество вопрос. В сложных задачах может быть больше одна целевая переменная или несколько промежуточных переменных, которые вы рассчитать.

               Теперь, зная целевые переменные, и ваш подход, вы можете собрать свой инструментарий математических выражения, используя принципы и ограничения из вашего подход, чтобы связать физические переменные из вашего диаграммы. Это первый раз, когда вы действительно начинаете смотреть для количественных отношений между переменными.

           ПЛАН РЕШЕНИЯ
          Прежде чем приступить к расчету ответ, найдите время, чтобы составить план. Обычно, когда законы физики выражаются в уравнении, уравнение является общим, универсальное заявление. Вы должны построить конкретные алгебраические уравнения, которые позволят вам рассчитать целевую переменную.

          • Определите, как уравнения в вашем наборе инструментов могут быть объединены, чтобы найти вашу целевую переменную. Начните с уравнение, содержащее целевую переменную.
          • Определите все неизвестные в этом уравнении.
          • Найдите в своем наборе инструментов уравнения, содержащие эти неизвестные.
          • Продолжайте этот процесс до тех пор, пока ваши уравнения не будут содержать новые неизвестные.
          • Пронумеруйте каждое уравнение для удобства.
          • В настоящее время не решайте уравнения численно.

              Часто специалисты по решению проблем начните с целевой переменной и работайте в обратном направлении, чтобы определить путь к ответу. Иногда единицы помогут вам найти правильный путь. Например, если вы ищете скорость, вы знаете, что ваш окончательный ответ должен быть в м/с.

              У вас есть решение, если ваш план много независимых уравнений, так как есть неизвестные. Если не, определите другие уравнения или проверьте план, чтобы убедиться, что он вероятно, что переменная будет исключена из ваших уравнений.

              Если у вас одинаковое количество уравнений и неизвестных, укажите порядок решения уравнений алгебраически для целевой переменной. Как правило, вы начинаете свой построение плана в конце и работа в обратном направлении первый шаг, то есть вы записываете уравнение, содержащее сначала целевая переменная.

          ВЫПОЛНИТЬ ПЛАН
          Теперь вы готовы выполнить план.

          • Выполните алгебраические вычисления в порядке, указанном в вашем плане.
          • Когда вы закончите, у вас должно быть одно уравнение с ваша целевая переменная изолирована с одной стороны и известна только количества с другой стороны.
          • Подставьте значения (числа с единицами измерения) в это итоговое уравнение.
          • Убедитесь, что юниты согласованы, чтобы они отменили правильно.

              Наконец, вычислите численный результат для целевая переменная (переменные). Убедитесь, что ваш окончательный ответ ясно человеку, который будет оценивать ваше решение.

              Чрезвычайно важно решить алгебраически, прежде чем вставлять какие-либо числовые ценности. Некоторые неизвестные количества могут аннулироваться, и вы не нужно знать их числовое значение. В некоторые сложные задачи может быть полезно вычислить промежуточные численные результаты в качестве проверки обоснованности вашего решение.

          ОЦЕНКА РЕШЕНИЯ
          Наконец, вы готовы оценить свои отвечать. Здесь вы должны использовать здравый смысл в отношении того, как реальный мир работает так же, как и те аспекты физического мира, вы узнали на уроке физики.

          • Имеют ли векторные величины как величину, так и направление?
          • Может ли кто-нибудь последовать вашему решению?
          • Является ли результат разумным и соответствует ли ваш опыт? Вспомните, например, что автомобили не едьте по шоссе со скоростью 300 миль в час. Если вы кладете более холодный предмет в горячую воду, вода охлаждается вниз, и температура объекта повышается.
          • Имеют ли единицы смысл? Скорость не измеряется, например, в кг/с.
          • Вы ответили на вопрос?

              По возможности рекомендуется внимательно прочитайте решение, особенно если оно оценивается вашим инструктором. Если ваша оценка предполагает вам, что ваш ответ неверный или необоснованный, сделайте заявление на этот счет и объяснить свои рассуждения.

          Дальнейшее чтение:

          Патрисия Хеллер, Рональд Кейт и Скотт Андерсон (1992), Обучение решению проблем через совместную группировку. Часть 1: Групповое и индивидуальное решение проблем, , США Журнал физики , Vol. 60, № 7, стр. 627-636.

          Патрисия Хеллер и Марк Холлабо (1992), Проблема обучения Решение через кооперативную группировку. Часть 2: Проектирование задач и структурирование групп, , США Журнал физики , Том. 60, № 7, стр. 637-644.

          How To — Physics — University of Wisconsin-Green Bay

          How To Work Physics Problems

          Решение любой физической задачи состоит из пяти ключевых шагов. Нажмите на любую из ссылок ниже, чтобы увидеть один шаг в контексте типа проблемы.

          1. Как определить ( , , , , , , , , ) проблема.
          2. Как нарисовать картинку для ( , , , , , , , , ) проблема.
          3. Как выбрать связь для ( , , , , , , , , ) проблема.
          4. Как решить ( , , , , , , , , ) проблема.
          5. Как понять результаты ( , , , , , , , , ) проблема.

          Прокрутите вниз для получения более подробной информации обо всех пяти шагах или щелкните ссылку «Как сделать» под каждым типом проблемы на странице «Проблемы», чтобы увидеть весь процесс в контексте.


          Общий подход к решению задач физики

          • 1. Определите проблему

            Чтобы определить тип вашей проблемы, подумайте о ключевой физике, лежащей в основе вашей проблемы. Особенности поверхности (в автомобиле, на склоне, на веревке, с трением или без, горизонтальная или вертикальная и т. д.) не влияют на то, как вы решаете задачу. Независимо от особенностей поверхности все задачи кинематики решаются одинаково. Точно так же все проблемы динамики решаются одинаково; все энергетические проблемы решаются одинаково; и т. д.

            В рамках процесса определения ключевой физики проблемы вам необходимо определить, какую систему следует учитывать. Другими словами, какой объект (или объекты) вам нужно отслеживать, чтобы ответить на поставленный перед вами вопрос. Система, которую вы выбираете для задачи сохранения импульса, часто сильно отличается от системы, которую вы выбрали бы для задачи силы, поэтому вопросы ключевой физики и системы, которые необходимо учитывать, взаимосвязаны друг с другом.

            Обратите внимание, что выявление проблемы является одновременно наиболее важным и наиболее часто пропускаемым этапом решения проблемы. Когда вы делаете домашнюю работу, вы обычно знаете тип задачи, потому что она назначается из определенной главы вашего текста. Кроме того, во многих книгах также указывается номер раздела, относящегося к каждой проблеме. Но когда придет время сдавать зачеты, а особенно выпускной экзамен, у вас не будет внешних сигналов. Несколько секунд, которые потребуются, чтобы четко сформулировать для себя, откуда вы знаете, какая у вас проблема (для каждой домашней задачи, над которой вы работаете), действительно окупятся как лучшим пониманием, так и лучшими оценками на экзаменах.

          • 2. Нарисуй картинку

            Каждому типу задач по физике соответствует определенный тип изображения. Рисунок, который вы рисуете, включает в себя всю информацию, необходимую для решения задачи (и, в идеале, только эту информацию) в формате, который напрямую связан с уравнением, которое вы будете использовать.

            Обратите внимание: как только вы определите проблему и нарисуете картинку, вы поместите все свое понимание физики на место. Остается только математическое решение ситуации, которую вы стилизовали на своей картинке.

          • 3. Выберите связь

            Во многих случаях, как только вы определили ключевую физику проблемы, которую необходимо решить, остается только одно уравнение, описывающее эту ключевую физику. В очень редких случаях (например, в кинематике) вам может потребоваться выбрать одно из нескольких соотношений.

            Обратите внимание, что могут возникать проблемы, требующие заполнения подуравнений по мере продвижения. Например, если вы работаете с законом сохранения энергии, вам нужно знать, что гравитационная потенциальная энергия выражается в mgh, а если вы работаете с выталкивающей силой, вам может понадобиться помнить, что плотность выражается в m/V. Однако вы поймете необходимость этих уравнений по мере того, как будете к ним приходить. Всегда начинайте с ключевой физики и соответствующего отношения.

          • 4. Решение проблемы

            Этот шаг обычно привлекает наибольшее внимание учащихся и заслуживает наименьшего внимания. Если вы нарисовали подходящую картинку, вам просто нужно поместить информацию с картинки в уравнение прямой пары и использовать уже известную математику, чтобы найти численное решение.

          • 5. Понимание результатов

            После того, как вы решите задачу, посмотрите на нее еще раз. Ваш ответ имеет смысл? Дало ли это поведение, которое вы интуитивно ожидали найти? Можете ли вы теперь сделать шаги, которые раньше вызывали у вас проблемы? Можете ли вы объяснить словами, что происходит? Если вы распознали тип задачи только по заголовку раздела в учебнике, определите информацию, которую вы будете использовать, чтобы распознать подобную задачу на выпускном экзамене. Несколько секунд, потраченных на понимание каждой проблемы, сэкономят вам время, поскольку вы столкнетесь с проблемами того же типа в будущем!

          См.
          эти шаги в контексте конкретного типа проблемы:
            • Работа с векторами
            • Проблемы определения
            • Проблемы кинематики
            • Проблемы с динамикой (силой)
            • Сохранение энергии
            • Сохранение импульса
            • Электричество и магнетизм
            • Схемы
            • Индукция

          1.7 Решение задач по физике

          Цели обучения

          К концу этого раздела вы сможете:

          • Описывать процесс разработки стратегии решения задач.
          • Объясните, как найти численное решение задачи.
          • Кратко опишите процесс оценки значимости численного решения задачи.

          Рис. 1.13 Навыки решения задач необходимы для успеха в физике. (кредит: «scui3asteveo»/Flickr)

          Навыки решения задач явно необходимы для успеха в количественном курсе физики. Что еще более важно, способность применять общие физические принципы, обычно представляемые уравнениями, к конкретным ситуациям является очень мощной формой знания. Это намного мощнее, чем запоминание списка фактов. Аналитические навыки и способность решать проблемы могут быть применены к новым ситуациям, в то время как список фактов не может быть достаточно длинным, чтобы охватить все возможные обстоятельства. Такие аналитические навыки полезны как для решения задач в этом тексте, так и для применения физики в повседневной жизни.

          Как вы, наверное, хорошо знаете, для решения проблем требуется определенный творческий подход и проницательность. Никакая жесткая процедура не работает каждый раз. Креативность и проницательность растут с опытом. С практикой основы решения проблем становятся почти автоматическими. Один из способов попрактиковаться — прорабатывать примеры текста для себя во время чтения. Другой вариант — проработать как можно больше задач в конце раздела, начиная с самых простых, чтобы укрепить уверенность, а затем переходя к более сложным. После того, как вы начнете заниматься физикой, вы увидите ее вокруг себя и сможете начать применять ее к ситуациям, с которыми сталкиваетесь за пределами классной комнаты, точно так же, как это делается во многих приложениях в этом учебнике.

          Хотя простого пошагового метода, подходящего для решения каждой проблемы, не существует, описанный ниже трехэтапный процесс облегчает решение проблемы и делает его более осмысленным. Три этапа — это стратегия, решение и значимость. Этот процесс используется в примерах по всей книге. Здесь мы рассмотрим каждый этап процесса по очереди.

          Стратегия

          Стратегия — это начальный этап решения проблемы. Идея состоит в том, чтобы точно выяснить, в чем заключается проблема, а затем разработать стратегию ее решения. Вот некоторые общие рекомендации для этого этапа:

          • Изучите ситуацию, чтобы определить, какие физические принципы задействованы . Часто бывает полезно сначала нарисовать простой эскиз . Вам часто нужно решить, какое направление является положительным, и отметить это на своем эскизе. Когда вы определили физические принципы, гораздо легче найти и применить уравнения, представляющие эти принципы. Хотя найти правильное уравнение очень важно, имейте в виду, что уравнения представляют собой физические принципы, законы природы и отношения между физическими величинами. Без концептуального понимания проблемы численное решение не имеет смысла.
          • Составьте список того, что дано или может быть выведено из заявленной проблемы (укажите «известные») . Многие проблемы сформулированы очень кратко и требуют некоторого изучения, чтобы определить, что известно. На этом этапе также может быть очень полезным рисование эскиза. Формальная идентификация известных имеет особое значение при применении физики к реальным ситуациям. Например, слово остановилось означает, что в этот момент скорость равна нулю. Кроме того, мы часто можем принять начальное время и положение за ноль, выбрав соответствующую систему координат.
          • Определите, что именно нужно определить в задаче (идентифицируйте неизвестные) . Особенно в сложных задачах не всегда очевидно, что нужно найти и в какой последовательности. Составление списка может помочь выявить неизвестные.
          • Определите, какие физические принципы могут помочь вам решить проблему . Поскольку физические принципы имеют тенденцию выражаться в виде математических уравнений, здесь может помочь список известных и неизвестных. Легче всего, если вы сможете найти уравнения, содержащие только одно неизвестное, то есть все остальные переменные известны, так что вы сможете легко найти неизвестное. Если уравнение содержит более одного неизвестного, то для решения задачи необходимы дополнительные уравнения. В некоторых задачах необходимо определить несколько неизвестных, чтобы получить наиболее необходимое. В таких задачах особенно важно помнить о физических принципах, чтобы не заблудиться в море уравнений. Возможно, вам придется использовать два (или более) разных уравнения, чтобы получить окончательный ответ.

          Решение

          Стадия решения — это когда вы делаете математику. Подставьте известные величины (вместе с их единицами измерения) в соответствующее уравнение и получите численное решение с единицами измерения . То есть проделайте алгебру, исчисление, геометрию или арифметику, необходимые для нахождения неизвестного из известного, убедившись, что в вычислениях используются единицы измерения. Этот шаг явно важен, потому что он дает числовой ответ вместе с его единицами измерения. Обратите внимание, однако, что этот этап составляет лишь одну треть всего процесса решения проблемы.

          Значение

          После выполнения математических расчетов на этапе решения проблемы возникает искушение подумать, что вы закончили. Но всегда помните, что физика — это не математика. Скорее, занимаясь физикой, мы используем математику как инструмент, помогающий нам понять природу. Итак, после получения числового ответа всегда следует оценивать его значимость:

          • Проверьте свои единицы измерения. Если единицы измерения ответа неверны, значит, была допущена ошибка, и вам следует вернуться к предыдущим шагам, чтобы найти ее. Один из способов найти ошибку — проверить все уравнения, которые вы вывели, на согласованность размеров. Однако имейте в виду, что правильные единицы измерения не гарантируют правильность числовой части ответа.
          • Проверьте ответ, чтобы убедиться, что он разумен. Имеет ли это смысл? Этот шаг чрезвычайно важен: – цель физики – точно описать природу. Чтобы определить, является ли ответ разумным, проверьте как его величину, так и его знак, в дополнение к его единицам измерения. Величина должна соответствовать приблизительной оценке того, какой она должна быть. Его также следует разумно сравнивать с величинами других величин того же типа. Знак обычно говорит вам о направлении и должен соответствовать вашим прежним ожиданиям. Ваше суждение улучшится по мере того, как вы будете решать больше физических задач, и у вас появится возможность делать более точные суждения о том, адекватно ли природа описана ответом на задачу. Этот шаг возвращает проблеме ее концептуальное значение. Если вы можете судить о том, является ли ответ разумным, у вас есть более глубокое понимание физики, чем просто возможность решить проблему механически.
          • Проверьте, не говорит ли ответ вам что-нибудь интересное. Что это значит? Это обратная сторона вопроса: есть ли смысл? В конечном счете, физика — это понимание природы, и мы решаем физические задачи, чтобы немного узнать о том, как работает природа. Поэтому, предполагая, что ответ действительно имеет смысл, вы всегда должны уделить время тому, чтобы посмотреть, говорит ли он вам что-то о мире, что вы находите интересным. Даже если ответ на эту конкретную проблему вам не очень интересен, как насчет метода, который вы использовали для ее решения? Можно ли адаптировать метод для ответа на интересующий вас вопрос? Во многих отношениях наука движется вперед, отвечая на подобные вопросы.

          Резюме

          В этой книге используются следующие три этапа процесса решения физических задач:

          • Стратегия : Определите, какие физические принципы задействованы, и разработайте стратегию их использования для решения задачи.
          • Решение : Выполните необходимые расчеты, чтобы получить численное решение с единицами измерения.
          • Значение : Проверьте решение, чтобы убедиться, что оно имеет смысл (правильные единицы измерения, разумная величина и знак), и оцените его значимость.

          Концептуальные вопросы

          Какая информация вам нужна, чтобы выбрать, какое уравнение или уравнения использовать для решения задачи?

          Что делать после получения числового ответа при решении задачи?

          Показать решение

          Дополнительные задачи

          Рассмотрим уравнение y = mt +b , где размерность y — длина, а размерность t — время, а m и b — константы. Каковы размеры и единицы СИ для (а) 9{5}\text{/}120, [/latex], где s — длина, а t — время. Каковы размеры и единицы СИ для (a) [латекса] {s}_{0}, [/латекса] (б) [латекса] {v}_{0}, [/латекса] (в) [латекса] {a}_{0}, [/latex] (d) [латекс] {j}_{0}, [/latex] (e) [латекс] {S}_{0}, [/latex] и ( е) в ?

          Показать решение

          (a) Автомобильный спидометр имеет погрешность 5%. Каков диапазон возможных скоростей, когда он показывает 90 км/ч? (b) Переведите этот диапазон в мили в час. Примечание. 1 км = 0,6214 мили.

          Марафонец преодолевает дистанцию ​​42,188 км за 2 часа 30 минут и 12 секунд. Неопределенность пройденного расстояния составляет 25 м, а прошедшего времени — 1 с. (a) Рассчитайте процент неопределенности расстояния. (b) Рассчитайте процент неопределенности в истекшем времени. в) Какова средняя скорость в метрах в секунду? г) Какова неопределенность средней скорости?

          Показать решение

          Размеры сторон небольшой прямоугольной коробки составляют 1,80 ± 0,1 см, 2,05 ± 0,02 см и 3,1 ± 0,1 см в длину. Вычислите его объем и погрешность в кубических сантиметрах.

          Когда в Соединенном Королевстве использовались неметрические единицы измерения, использовалась единица массы, называемая фунт-масса (lbm), где 1 фунт = 0,4539 кг. (a) Если существует погрешность в 0,0001 кг в единице массы фунта, какова ее неопределенность в процентах? (b) Исходя из этой неопределенности в процентах, какая масса в фунтах имеет погрешность в 1 кг при переводе в килограммы?

          а. 0,02%; б. 1×10 4 фунтов

          Длина и ширина прямоугольной комнаты составляют 3,955 ± 0,005 м и 3,050 ± 0,005 м. Вычислите площадь помещения и ее неопределенность в квадратных метрах.

          Автомобильный двигатель перемещает поршень с круглым поперечным сечением диаметром 7,500 ± 0,002 см на расстояние 3,250 ± 0,001 см для сжатия газа в цилиндре. а) На сколько уменьшился объем газа в кубических сантиметрах? б) Найдите неопределенность в этом объеме.

          Показать решение

          Задачи-вызовы

          Первая атомная бомба была взорвана 16 июля 1945 года на полигоне Тринити примерно в 200 милях к югу от Лос-Аламоса. В 1947 году правительство США рассекретило киноленту взрыва. По этой киноленте британский физик Г. И. Тейлор смог определить скорость, с которой рос радиус огненного шара от взрыва. Затем, используя пространственный анализ, он смог определить количество энергии, выделившейся при взрыве, что в то время было тщательно охраняемым секретом. Из-за этого Тейлор не публиковал свои результаты до 19 лет.{\text{−}c} [/latex] и что [латекс] [k]=1. [/latex]) (b) Анализируя данные о высокоэнергетических обычных взрывчатых веществах, Тейлор обнаружил, что полученная им формула кажется справедливой до тех пор, пока константа k имеет значение 1,03. По киноленте он смог определить многие значения r и соответствующие значения t . Например, он обнаружил, что через 25,0 мс огненный шар имел радиус 130,0 м. Используйте эти значения вместе со средней плотностью воздуха 1,25 кг/м 3 , для расчета начального энерговыделения детонации Тринити в джоулях (Дж). ( Подсказка : Чтобы получить энергию в джоулях, вам нужно убедиться, что все числа, которые вы подставляете, выражены в основных единицах СИ.) (c) Энергия, выделяемая при больших взрывах, часто указывается в единицах «тонны». тротила» (сокращенно «т тротила»), где 1 т тротила составляет около 4,2 ГДж. Переведите свой ответ на (б) в килотонны тротила (то есть кт тротила). Сравните свой ответ с быстрой и грязной оценкой в ​​10 кт в тротиловом эквиваленте, сделанной физиком Энрико Ферми вскоре после того, как он стал свидетелем взрыва с расстояния, которое считалось безопасным. (Сообщается, что Ферми сделал свою оценку, бросив несколько клочков бумаги прямо перед тем, как остатки ударной волны ударили его, и посмотрел, как далеко они были унесены ею.) 9{3}+\cdots , [/latex], где [latex] {a}_{n} [/latex] — безразмерные константы для всех [latex] n=0,1,2,\cdots [/latex] и x — аргумент функции. (Если вы еще не изучали степенные ряды в исчислении, просто доверьтесь нам.) Используйте этот факт, чтобы объяснить, почему требование, чтобы все члены уравнения имели одинаковые размерности, является достаточным для определения размерной согласованности. То есть на самом деле это подразумевает, что аргументы стандартных математических функций должны быть безразмерными, поэтому на самом деле нет необходимости делать это последнее условие отдельным требованием определения размерной согласованности, как мы это сделали в этом разделе.

      Добавить комментарий

      Ваш адрес email не будет опубликован. Обязательные поля помечены *